Vous êtes sur la page 1sur 41

POWERS AND FUCTIONS OF THE PRESIDENT

A. EXECUTIVE POWER (Art. VII, Secs. 1 and 17)


Marcos v Manglapus
177 SCRA 669 and 178 SCRA 760
FACTS:
Former President Marcos, after his and his family spent three year exile in Hawaii, USA, sought to return to
the Philippines. The call is about to request of Marcos family to order the respondents to ISSUE travel order
to them and to enjoin the petition of the President's decision to bar their return to the Philippines.

ISSUE: Whether or not, in the exercise of the powers granted by the Constitution, the President may
prohibit
the
Marcoses
from
returning
to
the
Philippines.
RULING:
Yes
According to Section 1, Article VII of the 1987 Constitution: "The executive power shall be vested in the
President of the Philippines." The phrase, however, does not define what is meant by executive power
although the same article tackles on exercises of certain powers by the President such as appointing
power during recess of the Congress (S.16), control of all the executive departments, bureaus, and
offices (Section 17), power to grant reprieves, commutations, and pardons, and remit fines and forfeitures,
after conviction by final judgment (Section 19),treaty making power (Section 21), borrowing
power (Section
20), budgetary
power(Section
22), informing
power (Section
23).
The Constitution may have grant powers to the President, it cannot be said to be limited only to the
specific powers enumerated in the Constitution. Whatever power inherent in the government that is
neither legislative nor judicial has to be executive.
Gonzalez v Hechanova
9 SCRA 230
DECISION
CONCEPCION, J p:
This is an original action for prohibition with preliminary injunction.
It is not disputed that on September 22, 1963, respondent Executive Secretary authorized the importation
of 67,000 tons of foreign rice to be purchased from private sources, and created a rice procurement
committee composed of the other respondents herein 1 for the implementation of said proposed
importation. Thereupon, or on September 25, 1963, herein petitioner, Ramon A. Gonzales a rice planter,
and president of the Iloilo Palay and Corn Planters Association, whose members are, likewise, engaged in
the production of rice and corn filed the petition herein, averring that, in making or attempting to make
said importation of foreign rice, the aforementioned respondents "are acting without jurisdiction or in
excess of jurisdiction", because Republic Act No. 3452 which allegedly repeals or amends Republic Act
No. 2207 explicitly prohibits the importation of rice and corn by "the Rice and Corn Administration
or any other government agency"; that petitioner has no other plain, speedy and adequate remedy in the
ordinary course of law; and that a preliminary injunction is necessary for the preservation of the rights of
the parties during the pendency of this case and to prevent the judgment therein from becoming
ineffectual. Petitioner prayed, therefore, that said petition be given due course; that a writ of preliminary
injunction be forthwith ISSUEd restraining respondents, their agents or representatives from implementing
the decision of the Executive Secretary to import the aforementioned foreign rice; and that, after due
hearing, judgment be rendered making said injunction permanent.
Forthwith, respondents were required to file their answer to the petition which they did, and petitioner's
prayer for a writ of preliminary injunction was set for hearing, at which both parties appeared and argued
orally. Moreover, a memorandum was filed, shortly thereafter, by the respondents. Considering, later on,
that the resolution of said incident may require some pronouncements that would be more appropriate in a
decision on the merits of the case, the same was set for hearing on the merits soon thereafter. The parties,
however, waived the right to argue orally, although counsel for respondents filed their memoranda.

I. Sufficiency of Petitioner's interest.


Respondents maintain that the status of petitioner as a rice planter does not give him sufficient interest to
file the petition herein and secure the relief therein prayed for. We find no merit in this pretense. Apart
from prohibiting the importation of rice and corn "by the Rice and Corn Administration or any other
government agency", Republic Act No. 3452 declares, in Section 1 thereof, that "the policy of the
Government" is to "engage in the purchase of these basic foods directly from those tenants, farmers,
growers, producers and landowners in the Philippines who wish to dispose of their products at a price that
will afford them a fair and just return for their labor and capital investment. . . ." Pursuant to this provision,
petitioner, as a planter with a rice land of substantial proportion, is entitled to a chance to sell to the
Government the rice it now seeks to buy abroad. Moreover, since the purchase of said commodity will have
to be effected with public funds mainly raised by taxation, and as a rice producer and landowner petitioner
must necessarily be a taxpayer, it follows that he has sufficient personality and interest to seek judicial
assistance with a view to restraining what he believes to be an attempt to unlawfully disburse said funds.
II. Exhaustion of administrative remedies.
Respondents assail petitioner's right to the reliefs prayed for because he "has not exhausted all
administrative remedies available to him before coming to court". We have already HELD, however, that
the principle requiring the previous exhaustion of administrative remedies is not applicable "where the
question in dispute is purely a legal one" , or where the controverted act is "patently illegal" or was
performed without jurisdiction or in excess of jurisdiction, or where the respondent is a department
secretary, whose acts as an alter-ego of the President bear the implied or assumed approval of the latter ,
unless actually disapproved by him, or where there are circumstances indicating the urgency of judicial
intervention. The case at bar falls under each one of the foregoing exceptions to the general rule.
Respondents' contention is, therefore, untenable.
III. Merits of petitioner's cause of action.
Respondents question the sufficiency of petitioner's cause of action upon the theory that the proposed
importation in question is not governed by Republic Act Nos. 2207 and 3452, but was authorized by the
President as commander-in-chief "for military stock pile purposes" in the exercise of his alleged authority
under Section 2 of Commonwealth Act No. 1; that in cases of necessity, the President "or his subordinates
may take such preventive measure for the restoration of good order and maintenance of peace"; and that,
as Commander-in-Chief of our armed forces, "the President . . . is duty-bound to prepare for the challenge
of threats of war or emergency without waiting for any special authority."
Regardless of whether Republic Act No. 3452 repeals Republic Act No. 2207, as contended by petitioner
herein on which our view need not be expressed we are unanimously of the opinion assuming that
said Republic Act No. 2207 is still in force that the two Acts are applicable to the proposed importation in
question because the language of said laws is such as to include within the purview
thereof all importations of rice and corn into the Philippines. Pursuant to Republic Act No. 2207, "it shall be
unlawful for any person, association, corporation or government agency to import rice and corn into any
point in the Philippines", although, by way of exception, it adds that "the President of the Philippines may
authorize the importation of these commodities through any government agency that he may designate",
if the conditions prescribed in Section 2 of said Act are present. Similarly, Republic Act No. 3452 explicitly
enjoins "the Rice and Corn Administration or any government agency" from importing rice and corn.
Respondents allege, however, that said provisions of Republic Acts Nos. 2207 and 3452, prohibiting the
importation of rice and corn by any "government agency", do not apply to importations "made by the
Government itself", because the latter is not a "government agency". This theory is devoid of merit. The
Department of National Defense and the Armed Forces of the Philippines, as well as respondents herein,
and each and every officer and employee of our Government, are government agencies and/or agents. The
applicability of said laws even to importations by the Government, as such, becomes more apparent when
we consider that:
1. The importation permitted in Republic Act No. 2207 is to be authorized by "the President of the
Philippines", and, hence, by or on behalf of the Government of the Philippines;
2. Immediately after enjoining the Rice and Corn Administration and any other government agency from
importing rice and corn, Section 10 of Republic Act No. 3452 adds " that the importation of rice and corn is

left to private parties upon payment of the corresponding taxes", thus indicating that only "private parties"
may import rice under its provisions; and
3. Aside from prescribing a fine not exceeding P10,000.00 and imprisonment of not more than five (5)
years for those who shall violate any provision of Republic Act No. 3452 or any rule and regulation
promulgated pursuant thereto, Section 15 of said Act provides that "if the offender is a public
official and/or employee", he shall be subject to the additional penalty specified therein. A public official is
an officer of the Government itself, as distinguished from officers or employees of instrumentalities of the
Government. Hence, the duly authorized acts of the former are those of the Government, unlike those of a
government instrumentality which may have a personality of its own, distinct and separate from that of
the Government, as such. The provisions of Republic Act No. 2207 are, in this respect, even more explicit.
Section 3 thereof provides a similar additional penalty for any "officer or employee of the Government"
who "violates, abets or tolerates the violation of any provision" of said Act. Hence, the intent to apply the
same to transactions made by the very government is patent.
Indeed, the restrictions imposed in said Republic Acts are merely additional to those prescribed
in Commonwealth Act No. 138, entitled "An Act to give native products and domestic entities the
preference in the purchase of articles for the Government." Pursuant to Section 1 thereof:
"The Purchase and Equipment Division of the Government of the Philippines and other officers and
employees of the municipal and provincial governments and the Government of the Philippines and of
chartered cities, boards, commissions, bureaus, departments, offices, agencies, branches, and bodies of
any description, including government-owned companies, authorized to requisition, purchase, or contract
or make disbursements for articles, materials, and supplies for public use, public buildings, or public works,
shall give preference to materials . . . produced . . . in the Philippines or in the United States, and to
domestic entities, subject to the conditions herein below specified." (Emphasis supplied.)
Under this provision, in all purchase by the Government, including those made by and/or for the armed
forces, preference shall be given to materials produced in the Philippines. The importation involved in the
case at bar violates this general policy of our Government, aside from the provisions of Republic Act Nos.
2207 and 3452.
The attempt to justify the proposed importation by invoking reasons of national security predicated
upon the "worsening situation in Laos and Vietnam", and "the recent tension created by the Malaysia
problem" and the alleged powers of the President as Commander-in-Chief of all armed forces in the
Philippines, under Section 2 of the National Defense Act (Commonwealth Act No. 1), overlooks the fact that
the protection of local planters of rice and corn in a manner that would foster and accelerate selfsufficiency in the local production of said commodities constitutes a factor that is vital to our ability to
meet a possible national emergency. Even if the intent in importing goods in anticipation of such
emergency were to bolster up that ability, the latter would, instead, be impaired if the importation were so
made as to discourage our farmers from engaging in the production of rice.
Besides, the stocking of rice and corn for purposes of national security and/or national emergency is within
the purview of Republic Act No. 3452. Section 3 thereof expressly authorizes the Rice and Corn
Administration "to accumulate stocks as a national reserve in such quantities as it may deem proper and
necessary to meet any contingencies". Moreover, it ordains that "the buffer stocks HELD as a national
reserve . . . be deposited by the Administration throughout the country under proper dispersal plans . . .
and maybe released only upon the occurrence of calamities or emergencies . . . (Emphasis supplied.)
Again, the provisions of Section 2 of Commonwealth Act No. 1, upon which respondents rely so much, are
not self-executory. They merely outline the general objectives of said legislation. The means for the
attainment of those objectives are subject to congressional legislation. Thus, the conditions under which
the services of citizens, as indicated in said Section 2, may be availed of, are provided for in Sections 3, 4
and 51 to 88 of said Commonwealth Act No. 1. Similarly, Section 5 thereof specifies the manner in which
resources necessary for our national defense may be secured by the Government of the Philippines, but
only "during a national mobilization" , which does not exist. Inferentially, therefore, in the absence of a
national mobilization, said resources shall be produced in such manner as Congress may by other Laws
provide from time to time. Insofar as rice and corn are concerned, Republic Act Nos. 2207 and 3452, and
Commonwealth are such laws.
Respondents cite Corwin in support of their pretense, but in vain. An examination of the work cited shows
that Corwin referred to the powers of the President during "war time" or when he has placed the country or
a part thereof under "martial law". Since neither condition obtains in the case at bar, said work merely
proves that respondents' theory, if accepted, would, in effect, place the Philippines under martial

law, without a declaration of the Executive to that effect. What is worse, it would keep us perpetually under
martial law.
It has been suggested that even if the proposed importation violated Republic Acts Nos. 2207 and 3452, it
should, nevertheless, be permitted because "it redounds to the benefit of the people". Salus populi est
suprema lex, it is said.
If there were a local shortage of rice, the argument might have some value. But the respondents, as
officials of this Government, have expressly affirmed again and again that there is no rice shortage. And
the importation is avowedly for stockpile of the Army not the civilian population.
But let us follow the respondents' trend of thought. It has a more serious implication that appears on the
surface. It implies that if an executive officer believes that compliance with a certain statute will not
benefit the people, he is at liberty to disregard it. That idea must be rejected we still live under a rule of
law.
And then, "the people" are either producers or consumers. Now as respondents explicitly admit
Republic Acts Nos. 2207 and 3452 were approved by the Legislature for the benefit of producers and
consumers, i.e., the people, it must follow that the welfare of the people lies precisely in
the compliance with said Acts.
It is not for respondent executive officers now to set their own opinions against that of the Legislature, and
adopt means or ways to set those Acts at naught. Anyway, those laws permit importation but under
certain conditions, which have not been, and should be complied with.
IV. The Contracts With Vietnam and Burma.
It is lastly contended that the Government of the Philippines has already entered into two (2) contracts for
the purchase of rice, one with the Republic of Vietnam, and another with the Government of Burma; that
these contracts constitute valid executive agreements under international law; that such agreements
became binding and effective upon signing thereof by representatives of the parties thereto; that in case
of conflict between Republic Acts Nos. 2207 and 3452 on the one hand, and the aforementioned contracts,
on the other, the latter should prevail, because, if a treaty and a statute are inconsistent with each other,
the conflict must be resolved under the American jurisprudence in favor of the one which is latest in
point of time; that petitioner herein assails the validity of acts of the executive relative to foreign relations
in the conduct of which the Supreme Court cannot interfere; and that the aforementioned contracts have
already been consummated, the Government of the Philippines having already paid the price of the rice
involved therein through irrevocable letters of credit in favor of the sellers of said commodity. We find no
merit in this pretense.
The Court is not satisfied that the status of said contracts as alleged executive agreements has been
sufficiently established. The parties to said contracts do not appear to have regarded the same as
executive agreements. But, even assuming that said contracts may properly be considered as executive
agreements, the same are unlawful, as well as null and void, from a constitutional viewpoint, said
agreements being inconsistent with the provisions of Republic Acts Nos. 2207 and 3452. Although the
President
may,
under
the
American
constitutional
system,
enter
into
executive
agreements without previous legislative authority, he may not, by executive agreement, enter into a
transaction which is prohibited by statutes enacted prior thereto. Under the Constitution, the main function
of the Executive is to enforce laws enacted by Congress. The former may not interfere in the performance
of the legislative powers of the latter, except in the exercise of his veto power. He may not defeat
legislative enactments that have acquired the status of laws, by indirectly repealing the same through an
executive agreement providing for the performance of the very act prohibited by said laws.
The American theory to the effect that, in the event of conflict between a treaty and a statute, the one
which is latest in point of time shall prevail, is not applicable to the case at bar, for respondents not only
admit, but, also, insist that the contracts adverted to are not treaties. Said theory may be justified upon
the ground that treaties to which the United States is signatory require the advice and consent of its
Senate, and, hence, of a branch of the legislative department. No such justification can be given as
regards executive agreements not authorized by previous legislation, without completely upsetting the
principle of separation of powers and the system of checks and balances which are fundamental in our
constitutional set up and that of the United States.

As regards the question whether an international agreement may be invalidated by our courts, suffice it to
say that the Constitution of the Philippines has clearly settled it in the affirmative, by providing, in Section
2 of Article VIII thereof, that the Supreme Court may not be deprived "of its jurisdiction to review, revise,
reverse, modify, or affirm on appeal, certiorari, or writ of error, as the law or the rules of court may
provide, final judgments and decrees of inferior courts in (1) All cases in which the constitutionality or
validity of any treaty, law, ordinance, or executive order or regulation is in question". In other words, our
Constitution authorizes the nullification of a treaty, not only when it conflicts with the fundamental
law, but, also, when it runs counter to an act of Congress.
The alleged consummation of the aforementioned contracts with Vietnam and Burma does not render this
case academic. Republic Act No. 2207 enjoins our Government not from entering into contracts for the
purchase of rice, but from importing rice, except under the conditions prescribed in said Act. Upon the
other, Republic Act No. 3452 has two (2) main features, namely; (a) it requires the Government to
purchase rice and corn directly from our local planters, growers or landowners; and (b) it
prohibits importations of rice by the Government, and leaves such importations to private parties. The
pivotal ISSUE in this case is whether the proposed importation which has not been consummated as yet
is legally feasible.
Lastly, a judicial declaration of illegality of the proposed importation would not compel our Government to
default in the performance of such obligations as it may have contracted with the sellers of the rice in
question, because, aside from the fact that said obligations may be complied with without importing the
commodity into the Philippines, the proposed importation may still be legalized by complying with the
provisions of the aforementioned laws.

Gonzales vs Henchova
9 SCRA 230
FACTS:
Exec. Secretary Hechanova authorised the importation of foreign rice to be purchased from private
sources. Gonzales filed a petition opposing the said implementation because RA No. 3542 which allegedly
repeals or amends RA No. 2207, prohibits the importation of rice and corn "by the Rice and Corn
Administration or any other government agency."
Respondents alleged that the importation permitted in RA 2207 is to be authorized by the President of the
Philippines, and by or on behalf of the Government of the Philippines. They add that after enjoining the
Rice and Corn administration and any other government agency from importing rice and corn, S. 10 of RA
3542 indicates that only private parties may import rice under its provisions. They contended that the
government has already constitute valid executive agreements with Vietnam and Burma, that in case of
conflict between RA 2207 and 3542, the latter should prevail and the conflict be resolved under the
American jurisprudence.
ISSUE: W/N the executive agreements may be validated in our courts.
RULING:
No. The Court is not satisfied that the status of said tracts as alleged executive agreements has been
sufficiently established. Even assuming that said contracts may properly considered as executive
agreements, the same are unlawful, as well as null and void, from a constitutional viewpoint, said
agreements being inconsistent with the provisions of Republic Acts Nos. 2207 and 3452. Although the
President may, under the American constitutional system enter into executive agreements without
previous legislative authority, he may not, by executive agreement, enter into a transaction which is
prohibited by statutes enacted prior thereto.
Under the Constitution, the main function of the Executive is to enforce laws enacted by Congress. He may
not interfere in the performance of the legislative powers of the latter, except in the exercise of his veto
power. He may not defeat legislative enactments that have acquired the status of law, by indirectly
repealing the same through an executive agreement providing for the performance of the very act
prohibited by said laws.

V. The writ of preliminary injunction.


The members of the Court have divergent opinions on the question whether or not respondents herein
should be enjoined from implementing the aforementioned proposed importation. However, the majority
favors the negative view, for which reason the injunction prayed for cannot be granted.
WHEREFORE, judgment is hereby rendered declaring that respondent Executive Secretary had and has no
power to authorize the importation in question; that he exceeded his jurisdiction in granting said authority;
that said importation is not sanctioned by law and is contrary to its provisions; and that, for lack of the
requisite majority, the injunction prayed for must be and is, accordingly, denied. It is so ordered.
DENR VS DENR EMPLOYEES
G.R. No. 149724 [Alter ego of the President, Qualified Political Agency Doctrine]
FACTS:
DENR Reg 12 Employees filed a petition for nullity of the memorandum order ISSUEd by the Regional Exec.
Director of DENR, directing the immediate transfer of the DENR 12 Regional Offices from Cotabato to
Koronadal City. The memorandum was ISSUEd pursuant to DENR Executive Order ISSUEd by the DENR
Secretary.
ISSUE: Whether or not DENR Secretary has the authority to reorganize the DENR Region 12 Office.
RULING: The qualified political agency doctrine, all executive and administrative organizations are
adjuncts of the Executive Department, and the acts of the Secretaries of such departments, performed and
promulgated in the regular course of business, are, unless disapproved or reprobated by the Chief
Executive, are presumptively the acts of the Chief Executive. It is corollary to the control power of the
President as provided for under Art. VII Sec. 17 of the 1987 Constitution: "The President shall have control
of all the executive departments, bureaus, and offices. He shall ensure that the laws be faithfully
executed."
In the case at bar, the DENR Secretary can validly reorganize the DENR by ordering the transfer of the
DENR XII Regional Offices from Cotabato City to Koronadal, South Cotabato. The exercise of this authority
by the DENR Secretary, as an alter ego, is presumed to be the acts of the President for the latter had not
expressly repudiated the same
BIRAOGO VS Philippine Truth Commission of 2010
GR No. 192935 & GR No. 19303, December 7, 2010
FACTS:
Pres. Aquino signed E. O. No. 1 establishing Philippine Truth Commission of 2010 (PTC) dated July 30, 2010.
PTC is a mere ad hoc body formed under the Office of the President with the primary task to investigate
reports of graft and corruption committed by third-level public officers and employees, their co-principals,
accomplices and accessories during the previous administration, and to submit its finding and
recommendations to the President, Congress and the Ombudsman. PTC has all the powers of an
investigative body. But it is not a quasi-judicial body as it cannot adjudicate, arbitrate, resolve, settle, or
render awards in disputes between contending parties. All it can do is gather, collect and assess evidence
of graft and corruption and make recommendations. It may have subpoena powers but it has no power to
cite people in contempt, much less order their arrest. Although it is a fact-finding body, it cannot determine
from such FACTS if probable cause exists as to warrant the filing of an information in our courts of law.
Petitioners asked the Court to declare it unconstitutional and to enjoin the PTC from performing its
functions. They argued that:
(a) E.O. No. 1 violates separation of powers as it arrogates the power of the Congress to create a public
office and appropriate funds for its operation.
(b) The provision of Book III, Chapter 10, Section 31 of the Administrative Code of 1987 cannot legitimize
E.O. No. 1 because the delegated authority of the President to structurally reorganize the Office of the
President to achieve economy, simplicity and efficiency does not include the power to create an entirely
new public office which was hitherto inexistent like the Truth Commission.

(c) E.O. No. 1 illegally amended the Constitution and statutes when it vested the Truth Commission with
quasi-judicial powers duplicating, if not superseding, those of the Office of the Ombudsman created under
the 1987 Constitution and the DOJ created under the Administrative Code of 1987.
(d) E.O. No. 1 violates the equal protection clause as it selectively targets for investigation and prosecution
officials and personnel of the previous administration as if corruption is their peculiar species even as it
excludes those of the other administrations, past and present, who may be indictable.
Respondents, through OSG, questioned the legal standing of petitioners and argued that:
1] E.O. No. 1 does not arrogate the powers of Congress because the Presidents executive power and
power of control necessarily include the inherent power to conduct investigations to ensure that laws are
faithfully executed and that, in any event, the Constitution, Revised Administrative Code of 1987, PD No.
141616 (as amended), R.A. No. 9970 and settled jurisprudence, authorize the President to create or form
such bodies.
2] E.O. No. 1 does not usurp the power of Congress to appropriate funds because there is no appropriation
but a mere allocation of funds already appropriated by Congress.
3] The Truth Commission does not duplicate or supersede the functions of the Ombudsman and the DOJ,
because it is a fact-finding body and not a quasi-judicial body and its functions do not duplicate, supplant
or erode the latters jurisdiction.
4] The Truth Commission does not violate the equal protection clause because it was validly created for
laudable purposes.
ISSUES:
1. WON the petitioners have legal standing to file the petitions and question E. O. No. 1;
2. WON E. O. No. 1 violates the principle of separation of powers by usurping the powers of Congress to
create
and
to
appropriate
funds
for
public
offices,
agencies
and
commissions;
3.
WON
E.
O.
No.
1
supplants
the
powers
of
the
Ombudsman
and
the
DOJ;
4. WON E. O. No. 1 violates the equal protection clause.
RULING:
The power of judicial review is subject to limitations, to wit: (1) there must be an actual case or
controversy calling for the exercise of judicial power; (2) the person challenging the act must have the
standing to question the validity of the subject act or issuance; otherwise stated, he must have a personal
and substantial interest in the case such that he has sustained, or will sustain, direct injury as a result of its
enforcement; (3) the question of constitutionality must be raised at the earliest opportunity; and (4) the
ISSUE of constitutionality must be the very lis mota of the case.
1. The petition primarily invokes usurpation of the power of the Congress as a body to which they belong
as members. To the extent the powers of Congress are impaired, so is the power of each member thereof,
since his office confers a right to participate in the exercise of the powers of that institution.
Legislators have a legal standing to see to it that the prerogative, powers and privileges vested by the
Constitution in their office remain inviolate. Thus, they are allowed to question the validity of any official
action which, to their mind, infringes on their prerogatives as legislators.
With regard to Biraogo, he has not shown that he sustained, or is in danger of sustaining, any personal and
direct injury attributable to the implementation of E. O. No. 1.
Locus standi is a right of appearance in a court of justice on a given question. In private suits, standing is
governed by the real-parties-in interest rule. It provides that every action must be prosecuted or
defended in the name of the real party in interest. Real-party-in interest is the party who stands to be
benefited or injured by the judgment in the suit or the party entitled to the avails of the suit.
Difficulty of determining locus standi arises in public suits. Here, the plaintiff who asserts a public right in
assailing an allegedly illegal official action, does so as a representative of the general public. He has to
show that he is entitled to seek judicial protection. He has to make out a sufficient interest in the
vindication of the public order and the securing of relief as a
citizen or taxpayer.
The person who impugns the validity of a statute must have a personal and substantial interest in the
case such that he has sustained, or will sustain direct injury as a result. The Court, however, finds reason
in Biraogos assertion that the petition covers matters of transcendental importance to justify the exercise

of jurisdiction by the Court. There are constitutional ISSUEs in the petition which deserve the attention of
this Court in view of their seriousness, novelty and weight as precedents
The Executive is given much leeway in ensuring that our laws are faithfully executed. The powers of the
President are not limited to those specific powers under the Constitution. One of the recognized powers of
the President granted pursuant to this constitutionally-mandated duty is the power to create ad hoc
committees. This flows from the obvious need to ascertain FACTS and determine if laws have been
faithfully executed. The purpose of allowing ad hoc investigating bodies to exist is to allow an inquiry into
matters which the President is entitled to know so that he can be properly advised and guided in the
performance of his duties relative to the execution and enforcement of the laws of the land.
2. There will be no appropriation but only an allotment or allocations of existing funds already
appropriated. There is no usurpation on the part of the Executive of the power of Congress to appropriate
funds. There is no need to specify the amount to be earmarked for the operation of the commission
because, whatever funds the Congress has provided for the Office of the President will be the very source
of the funds for the commission. The amount that would be allocated to the PTC shall be subject to existing
auditing rules and regulations so there is no impropriety in the funding.
3. PTC will not supplant the Ombudsman or the DOJ or erode their respective powers. If at all, the
investigative function of the commission will complement those of the two offices. The function of
determining probable cause for the filing of the appropriate complaints before the courts remains to be
with the DOJ and the Ombudsman. PTCs power to investigate is limited to obtaining FACTS so that it can
advise and guide the President in the performance of his duties relative to the execution and enforcement
of the laws of the land.
4. Court finds difficulty in upholding the constitutionality of Executive Order No. 1 in view of its apparent
transgression of the equal protection clause enshrined in Section 1, Article III (Bill of Rights) of the 1987
Constitution.
Equal protection requires that all persons or things similarly situated should be treated alike, both as to
rights conferred and responsibilities imposed. It requires public bodies and institutions to treat similarly
situated individuals in a similar manner. The purpose of the equal protection clause is to secure every
person within a states jurisdiction against intentional and arbitrary discrimination, whether occasioned by
the express terms of a statue or by its improper execution through the states duly constituted authorities.
There must be equality among equals as determined according to a valid classification. Equal protection
clause permits classification. Such classification, however, to be valid must pass the test of
reasonableness. The test has four requisites: (1) The classification rests on substantial distinctions; (2) It is
germane to the purpose of the law; (3) It is not limited to existing conditions only; and (4) It applies equally
to all members of the same class.
The classification will be regarded as invalid if all the members of the class are not similarly treated, both
as to rights conferred and obligations imposed.
Executive Order No. 1 should be struck down as violative of the equal protection clause. The clear mandate
of truth commission is to investigate and find out the truth concerning the reported cases of graft and
corruption during the previous administration only. The intent to single out the previous administration is
plain, patent and manifest.
Arroyo administration is but just a member of a class, that is, a class of past administrations. It is not a
class of its own. Not to include past administrations similarly situated constitutes arbitrariness which the
equal protection clause cannot sanction. Such discriminating differentiation clearly reverberates to label
the commission as a vehicle for vindictiveness and selective retribution. Superficial differences do not
make for a valid classification.
The PTC must not exclude the other past administrations. The PTC must, at least, have the authority to
investigate all past administrations.
The Constitution is the fundamental and paramount law of the nation to which all other laws must conform
and in accordance with which all private rights determined and all public authority administered. Laws that
do not conform to the Constitution should be stricken down for being unconstitutional.

WHEREFORE, the petitions are GRANTED. Executive Order No. 1 is hereby declared UNCONSTITUTIONAL
insofar as it is violative of the equal protection clause of the Constitution

Vinuya v Romulo
619 SCRA 533 (2010)
FACTS:
This is an original Petition for Certiorari under Rule 65 of the Rules of Court with an application for
issuance of a writ of preliminary mandatory injunction against the Office of the Executive Secretary,
Secretary of the DFA, the Secretary of the DOJ, and the OSG.
Petitioners are all members of the MALAYA LOLAS, a non-stock, non-profit organization registered with
SEC, established for the purpose of providing aid to the victims of rape by Japanese military forces in
Philippines during the Second World War.

the
the
the
the

Petitioners claim that since 1998, they have approached the Executive Department through the DOJ, DFA,
and OSG, requesting assistance in filing a claim against the Japanese officials and military officers who
ordered the establishment of the comfort women stations in the Philippines. But officials of the Executive
Department declined to assist the petitioners, and took the position that the individual claims of the
comfort women for compensation had already been fully satisfied by Japans compliance with the Peace
Treaty between the Philippines and Japan.
Hence, this petition where petitioners pray for this court to (a) declare that respondents committed grave
abuse of discretion amounting to lack or excess of discretion in refusing to espouse their claims for the
crimes against humanity and war crimes committed against them; and (b) compel the respondents to
espouse their claims for official apology and other forms of reparations against Japan before the
International Court of Justice (ICJ) and other international tribunals.
Respondents maintain that all claims of the Philippines and its nationals relative to the war were dealt with
in the San Francisco Peace Treaty of 1951 and the bilateral Reparations Agreement of 1956.
On January 15, 1997, the Asian Womens Fund and the Philippine government signed a Memorandum of
Understanding for medical and welfare support programs for former comfort women. Over the next five
years, these were implemented by the Department of Social Welfare and Development.
ISSUE: WON the Executive Department committed grave abuse of discretion in not espousing petitioners
claims for official apology and other forms of reparations against Japan.
RULING:
Petition lacks merit. From a Domestic Law Perspective, the Executive Department has the exclusive
prerogative to determine whether to espouse petitioners claims against Japan.
Political questions refer to those questions which, under the Constitution, are to be decided by the people
in their sovereign capacity, or in regard to which full discretionary authority has been delegated to the
legislative or executive branch of the government. It is concerned with ISSUEs dependent upon the
wisdom, not legality of a particular measure.
One type of case of political questions involves questions of foreign relations. It is well-established that
the conduct of the foreign relations of our government is committed by the Constitution to the executive
and legislativethe politicaldepartments of the government, and the propriety of what may be done in
the exercise of this political power is not subject to judicial inquiry or decision. are delicate, complex, and
involve large elements of prophecy. They are and should be undertaken only by those directly responsible
to the people whose welfare they advance or imperil.
But not all cases implicating foreign relations present political questions, and courts certainly possess the
authority to construe or invalidate treaties and executive agreements. However, the question whether the
Philippine government should espouse claims of its nationals against a foreign government is a foreign
relations matter, the authority for which is demonstrably committed by our Constitution not to the courts
but to the political branches. In this case, the Executive Department has already decided that it is to the
best interest of the country to waive all claims of its nationals for reparations against Japan in the Treaty of
Peace of 1951. The wisdom of such decision is not for the courts to question.

The President, not Congress, has the better opportunity of knowing the conditions which prevail in foreign
countries, and especially is this true in time of war. He has his confidential sources of information. He has
his agents in the form of diplomatic, consular and other officials.
The Executive Department has determined that taking up petitioners cause would be inimical to our
countrys foreign policy interests, and could disrupt our relations with Japan, thereby creating serious
implications for stability in this region. For the to overturn the Executive Departments determination would
mean an assessment of the foreign policy judgments by a coordinate political branch to which authority to
make that judgment has been constitutionally committed.
From a municipal law perspective, certiorari will not lie. As a general principle, where such an extraordinary
length of time has lapsed between the treatys conclusion and our consideration the Executive must be
given ample discretion to assess the foreign policy considerations of espousing a claim against Japan, from
the standpoint of both the interests of the petitioners and those of the Republic, and decide on that basis if
apologies are sufficient, and whether further steps are appropriate or necessary.
In the international sphere, traditionally, the only means available for individuals to bring a claim within
the international legal system has been when the individual is able to persuade a government to bring a
claim on the individuals behalf. By taking up the case of one of its subjects and by resorting to diplomatic
action or international judicial proceedings on his behalf, a State is in reality asserting its own right to
ensure, in the person of its subjects, respect for the rules of international law.
Within the limits prescribed by international law, a State may exercise diplomatic protection by whatever
means and to whatever extent it thinks fit, for it is its own right that the State is asserting. Should the
natural or legal person on whose behalf it is acting consider that their rights are not adequately protected,
they have no remedy in international law. All they can do is resort to national law, if means are available,
with a view to furthering their cause or obtaining redress. All these questions remain within the province of
municipal law and do not affect the position internationally.
Even the invocation of jus cogens norms and erga omnes obligations will not alter this analysis. Petitioners
have not shown that the crimes committed by the Japanese army violated jus cogens prohibitions at the
time the Treaty of Peace was signed, or that the duty to prosecute perpetrators of international crimes is
an erga omnes obligation or has attained the status of jus cogens.
The term erga omnes (Latin: in relation to everyone) in international law has been used as a legal term
describing obligations owed by States towards the community of states as a whole. Essential distinction
should be drawn between the obligations of a State towards the international community as a whole, and
those arising vis--vis another State in the field of diplomatic protection. By their very nature, the former
are the concern of all States. In view of the importance of the rights involved, all States can be HELD to
have a legal interest in their protection; they are obligations erga omnes.
The term jus cogens (literally, compelling law) refers to norms that command peremptory authority,
superseding conflicting treaties and custom. Jus cogens norms are considered peremptory in the sense
that they are mandatory, do not admit derogation, and can be modified only by general international
norms of equivalent authority
WHEREFORE, the Petition is hereby DISMISSED.

B. POWER OF CONTROL
(Sec. 17)
MONDANO V SILVOSA
97 Phil 143
Jose Mondano was the mayor of Mainit, Surigao. A complaint was filed against him for rape and
concubinage. The information reached the Assistant Executive Secretary who ordered the governor to
investigate the matter. Consequently, Governor Fernando Silvosa then summoned Mondano and the latter
appeared before him. Thereafter Silvosa suspended Mondano. Mondano filed a petition for prohibition
enjoining the governor from further proceeding.
In his defense, Silvosa invoked the Revised Administrative Code which provided that he, as part of the
executive and by virtue o the order given by the Assistant Executive Secretary, is with direct control,

direction, and supervision over all bureaus and offices under his jurisdiction . . . and to that end may
order the investigation of any act or conduct of any person in the service of any bureau or office under his
Department and in connection therewith may appoint a committee or designate an official or person who
shall conduct such investigations.
ISSUE: Whether or not the Governor, as agent of the Executive, can exercise the power of control over a
mayor.
HELD: No. (Note that Silvosa was asking as the agent of the Assistant Executive Secretary who ordered
him to investigate Mondano).
The Constitution provides:
The President shall have control of all the executive departments, bureaus, or offices, exercise general
supervision over all local governments as may be provided by law, and take care that the laws be faithfully
executed.
Under this constitutional provision the President has been invested with the power of control of all the
executive departments, bureaus, or offices, but not of all local governments over which he has been
granted only the power of general supervision as may be provided by law. The Department head
as agent of the President has direct control and supervision over all bureaus and offices under his
jurisdiction as provided for in section 79(c) of the Revised Administrative Code, but he does not have the
same control of local governments as that exercised by him over bureaus and offices under his jurisdiction.
Likewise, his authority to order the investigation of any act or conduct of any person in the service of any
bureau or office under his department is confined to bureaus or offices under his jurisdiction and does not
extend to local governments over which, as already stated, the President exercises only general
supervision as may be provided by law.
If the provisions of section 79 (c) of the Revised Administrative Code are to be construed as conferring
upon the corresponding department head direct control, direction, and supervision over all local
governments and that for that reason he may order the investigation of an official of a local government
for malfeasance in office, such interpretation would be contrary to the provisions of par 1, sec 10, Article 7,
of the 1935 Constitution.
In administrative law supervision means overseeing or the power or authority of an officer to see that
subordinate officers perform their duties. If the latter fail or neglect to fulfill them the former may take
such action or step as prescribed by law to make them perform their duties.
Control, on the other hand, means the power of an officer to alter or modify or nullify or set aside what a
subordinate officer had done in the performance of his duties and to substitute the judgment of the former
for that of the latter.
The Congress has expressly and specifically lodged the provincial supervision over municipal officials in the
provincial governor who is authorized to receive and investigate complaints made under oath against
municipal officers for neglect of duty, oppression, corruption or other form of maladministration of office,
and conviction by final judgment of any crime involving moral turpitude. And if the charges are serious,
he shall submit written charges touching the matter to the provincial board, furnishing a copy of such
charges to the accused either personally or by registered mail, and he may in such case suspend the
officer (not being the municipal treasurer) pending action by the board, if in his opinion the charge be one
affecting the official integrity of the officer in question. Sec 86 of the Revised Administrative Code adds
nothing to the power of supervision to be exercised by the Department Head over the administration of
municipalities.
In this case, the governor can only investigate Mondano for crimes relating to Mondanos office. If the issue
is not related to his office but involves a rime of moral turpitude (such as rape or concubinage as in this
case), there must first be a final conviction before a suspension may be issued. The point is, the governor
must suspend a mayor not because hes acting as an agent of the Executive but because of the power
granted him by the Revised Administrative Code.

VILLENA V SEC. OF INTERIOR


67 Phil 143
FACTS:
Division of Investigation of the DOJ, upon the request of the Secretary of the Interior, conducted an inquiry
into the conduct of the Villena, mayor of Makati, Rizal, as a result of which the latter was found to have
committed bribery, extortion, malicious abuse of authority ad unauthorized practice of the law profession.
The respondent recommended the suspension of Villena to the President of the Philippines, in which it was
verbally granted. The Secretary then suspended Villena from office. Villena filed a petition for preliminary
injunction against the Sec. to restrain him and his agents from proceeding with the investigation.
ISSUE:
Whether or not the Secretary of the Interior has jurisdiction or authority to suspend and order investigation
over Villena.
RULING:
The Secretary of Interior has the power to order investigation and to suspend Mayor Villena. As to the
power to order investigation, it was provided in Section 79 (C) of RAC that Department of Interior was
given the authority to supervise bureaus and offices under its jurisdiction. This was interpreted in relation
to Section 86 of the same Code which granted the said Department of executive supervision over
administration of provinces, municipalities and other political subdivisions. This supervision covers the
power to order investigation because supervision implies authority to inquire into FACTS and conditions in
order to render power real and effective.However, unlike this power to order investigation, the power to
suspend a mayor was not provided in any law. There was no express grant of authority to the Secretary of
Interior to suspend a Mayor. Nevertheless, Section 2188 of the Administrative Code granted the provincial
governor the power of suspension. Yet this did not mean that the grant precluded the Secretary of Interior.
The Doctrine of Qualified Political Agency which provides that the acts of the department secretaries,
performed and promulgated in the regular course of business, are, unless disapproved or reprobated by
the President, presumptively the acts of the President. The power to suspend may be exercised by the
President. It follows that the heads of the Department under her may also exercise the same, unless the
law required the President to act personally or that situation demanded him so, because the heads of the
departments are assistants and agents of the President.
FREE TEL WORKERS V MINISTER OF LABOR
108 SCRA 75
FACTS:
In 1981, there was an ongoing labor dispute between the Free Telephone Workers Union (the Union) and
the Philippine Long Distance Company. Eventually, the Minister of Labor (Blas Ople) assumed jurisdiction
over the ISSUE pursuant to Article 264 of the Labor Code. The Union assailed the provisions of Article 264
as it averred that it is an undue delegation of power by Congress to the Minister of Labor. They averred
that by granting discretion to the Minister of Labor to whether or not refer a labor dispute for compulsory
arbitration to the National Labor Relations Commission, it also effectively granted the Minister to make or
unmake the law on free collective bargaining.
ISSUE: Whether or not such provision is an undue delegation of power.
HELD: No. In the first place, this ISSUE is not yet ripe for adjudication as the Minister of Labor was yet to
take on the entirety of the case. There is still no ground to rule that there is an unconstitutional application
of the law.
The Union failed to make out a case of undue delegation of legislative power. There could be, however, an
unconstitutional application. For while the Constitution allows compulsory arbitration, it must be stressed
that the exercise of such competence cannot ignore the basic fundamental principle and state policy that
the state should afford protection to labor. But as to whether or not there is an unconstitutional application
of the law, that is yet to be determined since the Minister of Labor has not yet made a factual
determination of the labor dispute in ISSUE.

There is no undue delegation in this case. The law in ISSUE is complete and it set a sufficient standard. The
law cannot be any clearer, the coverage being limited to strikes or lockouts adversely affecting the
national interest.

Banda v Ermita
GR No. 166620, Arpil 20, 2010
FACTS:
President GMA ISSUEd Executive Order No. 378 on 2004 amending Section 6 of Executive Order No. 285by,
inter alia, removing the exclusive jurisdiction of the NPO (National Printing Office) over the printing
services requirements of government agencies and instrumentalities. Pursuant to Executive Order No. 378,
government agencies and instrumentalities are allowed to source their printing services from the private
sector through competitive bidding, subject to the condition that the services offered by the private
supplier be of superior quality and lower in cost compared to what was offered by the NPO. Executive
Order No. 378 also limited NPOs appropriation in the General Appropriations Act to its income. Perceiving
Executive Order No. 378 as a threat to their security of tenure as employees of the NPO, petitioners now
challenge its constitutionality, contending that: (1) it is beyond the executive powers of President Arroyo to
amend or repeal Executive Order No. 285 ISSUEd by former President Aquino when the latter still exercised
legislative powers; and (2) Executive Order No. 378 violates petitioners security of tenure, because it
paves the way for the gradual abolition of the NPO.
ISSUE: Whether EO 378 is constitutional.
HELD
: YES
J. Leonardo-de Castro
. It is a well-settled principle in jurisprudence that the President has the power to reorganize the offices and
agencies in the executive department in line with the Presidents constitutionally granted power of control
over executive offices and by virtue of previous delegation of the legislative power to reorganize executive
offices under existing statutes. Executive Order No. 292 or the Administrative Code of 1987 gives the
President continuing authority to reorganize and redefine the functions of the Office of the
President. Section 31, Chapter 10, Title III, Book III of the said Code, is explicit: The President
, subject to the policy in the Executive Office and in order to achieve simplicity, economy and
efficiency, shall have continuing authority to reorganize the administrative structure of the
Office of the President
.It is undisputed that the NPO, as an agency that is part of the Office of the Press Secretary (which in
various times has been an agency directly attached to the Office of the Press Secretary or as an agency
under the Philippine Information Agency), is part of the Office of the President. To be very clear, this
delegated legislative power to reorganize pertains only to the Office of the President and the departments,
offices and agencies of the executive branch and does not include the Judiciary, the Legislature or the
constitutionally-created or mandated bodies. Moreover, it must be stressed that the exercise by the
President of the power to reorganize the executive department must be in accordance with the
Constitution, relevant laws and prevailing jurisprudence.
J. Carpio:
RA 9184 mandates the conduct of competitive bidding in all the procurement activities of the government
including the acquisition of items, supplies, materials, and general support services x x x which may be
needed in the transaction of the public businesses or in the pursuit of any government x x x activity save
for limited transactions. By opening governments procurement of standard and accountable forms to
competitive bidding (except for documents crucial to the conduct of clean elections which has to be
printed solely by government), EO 378 merely implements RA 9184s principle of promoting
competitiveness by extending equal opportunity to enable private contracting parties who are eligible and
qualified to participate in public bidding
C. POWER OF GENERAL SUPERVISION over LGUS

Art. X, Sec 4, 16
Ganzon vs. CA
200 SCRA 271
FACTS:
Ganzon, after having been Issued three successive 60-day of suspension order by Secretary of Local
Government, filed a petition for prohibition with the CA to bar Secretary Santos from implementing the said
orders. Ganzon was faced with 10 administrative complaints on various charges on abuse of authority and
grave misconduct.
ISSUE:
Whether or not the Secretary of Local Government (as the alter ego of the President) has the authority to
suspend and remove local officials.

RULING:
The Constitution did nothing more, and insofar as existing legislation authorizes the President (through the
Secretary of Local Government) to proceed against local officials administratively, the Constitution
contains no prohibition. The Chief Executive is not banned from exercising acts of disciplinary authority
because she did not exercise control powers, but because no law allowed her to exercise disciplinary
authority.
In those case that this Court denied the President the power (to suspend/remove) it was not because that
the President cannot exercise it on account of his limited power, but because the law lodged the power
elsewhere. But in those cases in which the law gave him the power, the Court, as in Ganzon v. Kayanan,
found little difficulty in sustaining him.
We reiterate that we are not precluding the President, through the Secretary of Interior from exercising a
legal power, yet we are of the opinion that the Secretary of interior is exercising that power oppressively,
and needless to say, with a grave abuse of discretion.
As we observed earlier, imposing 600 days of suspension which is not a remote possibility Mayor Ganzon is
to all intents and purposes, to make him spend the rest of his term in inactivity. It is also to make, to all
intents and purposes, his suspension permanent.
DADOLE V COA
GR No. 125350, December 3, 2002
FACTS:
Acting on the DBM's Local Budget Circular No. 55, the Mandaue City Auditor ISSUEd notices of
disallowances to RTC and MTC Judges, in excess of the amount (maximum of P1000 and P700 in provinces
and cities and municipalities, respectively) authorized by said circular. The additional monthly allowances
of the judges shall be reduced to P1000 each. They were also asked to reimbursed the amount they
received in excess of P1000 from the last six months.
ISSUE: Whether or not Local Budget Circular No. 55 void for going beyond the supervisory powers of the
President.
RULING:
Yes. Although the Constitution guarantees autonomy to local government units, the exercise of local
autonomy remains subject to the power of control by Congress and the power of supervision by the
President. Sec 4 Art X of 1987 Constitution: "The President of the Philippines shall exercise general
supervision over local governments. x x x" The said provision has been interpreted to exclude the power of
control.
The members of the Cabinet and other executive officials are merely alter egos of the President. As such,
they are subject to the power of control of the President; he will see to it that the local governments or
their officials were performing their duties as provided by the Constitution and by statutes, at whose will
and behest they can be removed from office; or their actions and decisions changed, suspended or
reversed. They are subject to the President's supervision only, not control, so long as their acts are

exercised within the sphere of their legitimate powers. The President can only interfere in the affairs and
activities of a LGU if he or she finds that the latter has acted contrary to law. This is the scope of the
President's supervisory powers over LGUs
D. POWER OF APPOINTMENTS
GPI VS SPRINGER
50 Phil 259
FACTS:
Rodolfo Ganzon was the then mayor of Iloilo City. 10 complaints were filed against him on grounds of
misconduct and misfeasance of office. The Secretary of Local Government issued several suspension
orders against Ganzon based on the merits of the complaints filed against him hence Ganzon was facing
about 600 days of suspension. Ganzon appealed the issue to the CA and the CA affirmed the suspension
order by the Secretary. Ganzon asserted that the 1987 Constitution does not authorize the President nor
any of his alter ego to suspend and remove local officials; this is because the 1987 Constitution supports
local autonomy and strengthens the same. What was given by the present Constitution was mere
supervisory power.
ISSUE: Whether or not the Secretary of Local Government, as the Presidents alter ego, can suspend and
or remove local officials.
HELD: Yes. Ganzon is under the impression that the Constitution has left the President mere supervisory
powers, which supposedly excludes the power of investigation, and denied her control, which allegedly
embraces disciplinary authority. It is a mistaken impression because legally, supervision is not
incompatible with disciplinary authority.
The SC had occasion to discuss the scope and extent of the power of supervision by the President over
local government officials in contrast to the power of control given to him over executive officials of our
government wherein it was emphasized that the two terms, control and supervision, are two different
things which differ one from the other in meaning and extent. In administration law supervision means
overseeing or the power or authority of an officer to see that subordinate officers perform their duties. If
the latter fail or neglect to fulfill them the former may take such action or step as prescribed by law to
make them perform their duties.
Control, on the other hand, means the power of an officer to alter or modify or nullify of set aside what a
subordinate officer had done in the performance of his duties and to substitute the judgment of the former
for that of the latter. But from this pronouncement it cannot be reasonably inferred that the power of
supervision of the President over local government officials does not include the power of investigation
when in his opinion the good of the public service so requires.
The Secretary of Local Government, as the alter ego of the president, in suspending Ganzon is exercising a
valid power. He however overstepped by imposing a 600 day suspension.

Rufino vs Endriga
G.R. No. 139554, July 21, 2006
FACTS:
On 25 June 1966, then President Ferdinand E. Marcos issued Executive Order No. 30 (EO 30) creating the
Cultural Center of the Philippines as a trust governed by a Board of Trustees of seven members to preserve
and promote Philippine culture.
On 5 October 1972, or soon after the declaration of Martial Law, President Marcos issued PD 15, the
CCPs charter, which converted the CCP under EO 30 into a non-municipal public corporation free from the
pressure or influence of politics. PD 15 increased the members of CCPs Board from seven to nine

trustees. Later, Executive Order No. 1058, issued on 10 October 1985, increased further the trustees to
11.
After the People Power Revolution in 1986, then President Corazon C. Aquino asked for the courtesy
resignations of the then incumbent CCP trustees and appointed new trustees to the Board. Eventually,
during the term of President Fidel V. Ramos, the CCP Board included Endriga, Lagdameo,
Sison, Potenciano, Fernandez, Lenora A. Cabili (Cabili), and Manuel T. Maosa (Maosa).
On 22 December 1998, then President Joseph E. Estrada appointed seven new trustees to the CCP
Board for a term of four years to replace the Endriga group as well as two other incumbent trustees. The
seven new trustees were:
1.
2.
3.
4.
5.
6.
7.

Armita B. Rufino
Zenaida R. Tantoco
Federico Pascual
Rafael Buenaventura
Lorenzo Calma
Rafael Simpao, Jr.
Freddie Garcia
-

President, vice Baltazar N. Endriga


Member, vice Doreen Fernandez
Member, vice Lenora A. Cabili
Member, vice Manuel T. Maosa
Member, vice Ma. Paz D. Lagdameo
Member, vice Patricia C. Sison
Member, vice Irma Ponce-Enrile Potenciano

Except for Tantoco, the Rufino group took their respective oaths of office and assumed the performance
of their duties in early January 1999.
On 6 January 1999, the Endriga group filed a petition for quo warranto before this Court questioning
President Estradas appointment of seven new members to the CCP Board. The Endriga group alleged
that under Section 6(b) of PD 15, vacancies in the CCP Board shall be filled by election by a vote of a
majority of the trustees held at the next regular meeting x x x. In case only one trustee survive[s], the
vacancies shall be filled by the surviving trustee acting in consultation with the ranking officers of the
[CCP]. The Endriga group claimed that it is only when the CCP Board is entirely vacant may the President
of the Philippines fill such vacancies, acting in consultation with the ranking officers of the CCP.
The Endriga group asserted that when former President Estrada appointed the Rufino group, only one seat
was vacant due to the expiration of Maosas term. The CCP Board then had 10 incumbent trustees.
The Endriga group refused to accept that the CCP was under the supervision and control of the
President. The Endriga group cited Section 3 of PD 15, which states that the CCP shall enjoy autonomy of
policy and operation x x x.
On 14 May 1999, the Court of Appeals granted the quo warranto petition. The Court of Appeals declared
the Endriga group lawfully entitled to hold office as CCP trustees. On the other hand, the appellate courts
Decision ousted the Rufino group from the CCP Board.
In their motion for reconsideration, the Rufino group asserted that the law could only delegate to the CCP
Board the power to appoint officers lower in rank than the trustees of the Board. The law may not validly
confer on the CCP trustees the authority to appoint or elect their fellow trustees, for the latter would
be officers of equal rank and not of lower rank. Section 6(b) of PD 15 authorizing the CCP trustees to
elect their fellow trustees should be declared unconstitutional being repugnant to Section 16, Article VII of
the 1987 Constitution allowing the appointment only of officers lower in rank than the appointing
power.
On 3 August 1999, the Court of Appeals denied the Rufino groups motion for reconsideration. The Court of
Appeals also denied the Endriga groups motion for immediate execution of the 14 May 1999 Decision.
Hence, the instant consolidated petitions.
ISSUE: Whether or not Sec. 6 (b) of PD 15 is constitutional and CCP trustees have the authority to appoint
and elect their fellow trustees when there is vacancy.
RULING:
NO. The SC ruled that Sec. 6 (b) and (c) of PD 15 as amended which authorizes the remaining trustees to
fill by election vacancies in the Board of Trustees of CCP is unconstitutional.

Section 6(b) and (c) of PD 15, which authorizes the trustees of the CCP Board to fill vacancies in the Board,
runs afoul with the Presidents power of control under Section 17, Article VII of the 1987 Constitution. The
intent of Section 6(b) and (c) of PD 15 is to insulate the CCP from political influence and pressure,
specifically from the President. Section 6(b) and (c) of PD 15 makes the CCP a self-perpetuating entity,
virtually outside the control of the President. Such a public office or board cannot legally exist under the
1987 Constitution.
Section 3 of PD 15, as amended, states that the CCP shall enjoy autonomy of policy and operation x x
x. This provision does not free the CCP from the Presidents control, for if it does, then it would be
unconstitutional. This provision may give the CCP Board a free hand in initiating and formulating policies
and undertaking activities, but ultimately these policies and activities are all subject to the Presidents
power of control.
The CCP is part of the Executive branch. No law can cut off the Presidents control over the CCP in the
guise of insulating the CCP from the Presidents influence. By stating that the President shall have control
of all the executive x x x offices, the 1987 Constitution empowers the President not only to influence
but even to control all offices in the Executive branch, including the CCP. Control is far greater than,
and subsumes, influence.

SARMIENTO VS MISON
156 SCRA 549
FACTS:
Mison was appointed as the Commissioner of the Bureau of Customs and Carague as the Secretary of the
Department of Budget, without the confirmation of the Commission on Appointments. Sarmiento assailed
the appointments as unconstitutional by reason of its not having been confirmed by CoA.
ISSUE: Whether or not the appointment is valid.
RULING:
Yes. The President acted within her constitutional authority and power in appointing Salvador Mison,
without submitting his nomination to the CoA for confirmation. He is thus entitled to exercise the full
authority and functions of the office and to receive all the salaries and emoluments pertaining thereto.
Under Sec 16 Art. VII of the 1987 Constitution, there are 4 groups of officers whom the President shall
appoint:
1st, appointment of executive departments and bureaus heads, ambassadors, other public ministers,
consuls, officers of the armed forces from the rank of colonel or naval captain, and other officers with the
consent and confirmation of the CoA.
2nd, all other Government officers whose appointments are not otherwise provided by law;
3rd those whom the President may be authorized by the law to appoint;
4th, low-ranking officers whose appointments the Congress may by law vest in the President alone.
First group of officers is clearly appointed with the consent of the Commission on Appointments.
Appointments of such officers are initiated by nomination and, if the nomination is confirmed by the
Commission on Appointments, the President appoints.
2nd, 3rd and 4th group of officers are the present bone of contention. By following the accepted rule in
constitutional and statutory construction that an express enumeration of subjects excludes others not
enumerated, it would follow that only those appointments to positions expressly stated in the first group
require the consent (confirmation) of the Commission on Appointments.
It is evident that the position of Commissioner of the Bureau of Customs (a bureau head) is not one of
those within the first group of appointments where the consent of the Commission on Appointments is
required. The 1987 Constitution deliberately excluded the position of "heads of bureaus" from
appointments that need the consent (confirmation) of the Commission on Appointments.

CONCEPTION-BAUTISTA VS SALONGA

172 SCRA 549


FACTS:
In August 1987, then President Corazon Aquino designated Mary Concepcion-Bautista as the Acting
Chairwoman of Commission on Human Rights. In December 1987, Cory made the designation of Bautista
permanent. Bautista then took her oath of office.
Later however, Bautista received a letter from the Commission on Appointments (COA) requiring her to
submit certain documents for her qualification and for confirmation by the COA. Bautista then wrote a
letter to the COA Chairman, Senate President Jovito Salonga, and she explained that her position as
chairwoman of the CHR does not require confirmation by the COA as laid down in the case of Sarmiento vs
Mison.
Meanwhile, pending the issue of Bautistas appointment with the COA, Cory designated Hesiquio Mallilin as
the acting chairman of the CHR.
In 1989, the COA finally disapproved the appointment of Bautista. COA considered Bautistas appointment
as ad interim.
Bautista went to the Supreme Court and questioned COAs actions. She impleaded Mallillin. Mallillin on his
part invoked Executive Order No. 163-A which provided that the appointment of the CHR chair is at the
pleasure of the president. Hence, since Cory left the issue with the COA and the latter decided not to
confirm Bautista, Mallillin should be allowed to take his seat as chairman of the CHR.

ISSUE: Whether or not Bautistas appointment is subject to COAs confirmation.

HELD: No. The appointment of the Chairman and Members of the CHR is not specifically provided for in
the Constitution itself, unlike the Chairmen and Members of the Civil Service Commission, the Commission
on Elections and the Commission on Audit, whose appointments are expressly vested by the Constitution
in the President with the consent of the COA. The President appoints the Chairman and Members of the
CHR pursuant to the second sentence in Sec 16, Art. 7, that is, without the confirmation of the COA
because they are among the officers of government whom he (the President) may be authorized by law to
appoint. The law which authorizes the president to make appointments to the CHR is Executive Order
No. 163.
The act of Cory submitting Bautistas appointment to the COA for confirmation is merely political in nature
and it has no basis in law or in the constitution. Appointment to the CHR should be made without the
participation of the COA. Thus, Corys act of submitting the appointment of Bautista to the CHR is
done without or in excess of jurisdiction.
Even assuming arguendo that the President can submit such appointment to the COA for the latters
approval or rejection, such submission is not valid because at the time of submission, the office of the
chairman (chairwoman) of the CHR is not vacant as at that time, Bautista already took her oath and was
the incumbent CHR chairperson.
There is also no basis for the COA to consider Bautistas appointment as ad interim. Since the position of
chairman and members of the CHR are not subject to COA confirmation, all appointments to the CHR are
always permanent and cannot be ad interim.

Anent the argument of Mallillin that EO 163-A provides that the chairman and members of the CHR may be
removed at the pleasure of the president, the same is not valid. Thus, EO 163-A is unconstitutional. Note
that the earlier EO 163 provides that the chairman and the members of the CHR shall have a term of 7
years. The Chairman and the Commissioners of the CHR cannot be removed at the pleasure of the
president for it is guaranteed that they must have a term of office. They can only be removed upon cause
and with the observance of due process.

QUINTOS-DELES VS COA
177 SCRA 160
FACTS:
- August 27, 1987: President Cory Aquino appointed petitioner Mary Concepcion Bautista as permanent
Chairman of the Commission on Human Rights (CHR).
- December 22, 1988: Bautista took her oath of office to Chief Justice Marcelo Fernan and immediately
acted as such.
- January 9, 1989: The Secretary of the Commission on Appointments (CoA) wrote a letter to Bautista
requesting for her presence along with several documents at the office of CoA on January 19. Bautista
refused to be placed under CoA's review.
- Bautista filed a petition with the Supreme Court.
- While waiting for the progress of the case, President Aquino appointed Hesiquio R. Mallillin as "Acting
Chairman of the Commission on Human Rights" but he was not able to sit in his appointive office because
of Bautista's refusal to surrender her post.
- Malilin invoked EO 163-A which provides that the tenure of the Chairman and the Commissioners of the
CHR should be at the pleasure of the President thus stating that Bautista shall be subsequently removed as
well.
Issues:
WON the President's appointment is considered constitutional.
WON or not Bautista's appointment is subject to CoA's confirmation.
WON or not President should extend her appointment on January 14, 1989.
Held:
Sec. 16, Art. VII of the 1987 Constitution provides:
The President shall nominate and, with the consent of the Commission on Appointments, appoint the
heads of the executive departments, ambassadors, other public ministers and consuls, or officers of the
armed forces from the rank of colonel or naval captain, and other officers whose appointments are vested
in him in this Constitution. He shall also appoint all other officers of the Government whose appointments
are not otherwise provided for by law, and those whom he may be authorized by law to appoint.
The Congress may, by law, vest the appointment of other officers lower in rank in the President alone, in
the courts, or in the heads of the departments, agencies, commissions or boards. The President shall have
the power to make appointments during the recess of the Congress, whether voluntary or compulsory, but
such appointments shall be effective only until disapproval by the Commission on Appointments or until
the next adjournment of the Congress.
The Court held that it is within the authority of the President, vested upon her by the Constitution, that she
appoint Executive officials. The second sentence of the provision Section 16, Article VII provides that the
President is authorized by law to appoint, without confirmation of CoA, several government officials. The
position of Chairman of CHR is not among the positions mentioned in the first sentence of Sec. 16, Art VII
of the 1987 Constitution, which provides the appointments which are to be made with the confirmation of
CoA. It therefore follows that the appointment of the Chairman of CHR by the President is to be made and
finalized even without the review or participation of CoA. Bautista's appointment as the Chairman of CHR,
therefore, was already a completed act on the day she took her oath as the appointment was finalized
upon her acceptance, expressly stated in her oath.

Furthermore, the Court held that the provisions of EO 163-A is unconstitutional and thus cannot be invoked
by Mallillin. The Chairman of CHR cannot be removed at the pleasure of the President for it is
constitutionally guaranteed that they must have a term of office.
To hold, as the Court holds, that petitioner Bautista is the lawful incumbent of the office of Chairman of the
Commission on Human Rights by virtue of her appointment, as such, by the President on 17 December
1988, and her acceptance thereof, is not to say that she cannot be removed from office before the
expiration of her seven (7) year term. She certainly can be removed but her removal must be for cause
and with her right to due process properly safeguarded.
It is to the credit of the President that, in deference to the rule of law, after petitioner Bautista had
elevated her case to this Tribunal, Her Excellency merely designated an Acting Chairman for the
Commission on Human Rights (pending decision in this case) instead of appointing another permanent
Chairman. The latter course would have added only more legal difficulties to an already difficult situation.
Petitioner Bautista is declared to be, as she is, the duly appointed Chairman of the Commission on Human
Rights and the lawful incumbent thereof, entitled to all the benefits, privileges and emoluments of said
office. The temporary restraining order heretofore issued by the Court against respondent Mallillin
enjoining him from dismissing or terminating personnel of the Commission on Human Rights is made
permanent.

CALDERON VS CARALE
208 SCRA 254
FACTS:
In 1989, Republic Act No. 6715 was passed. This law amended PD 442 or the Labor Code. RA 6715 provides
that the Chairman, the Division Presiding Commissioners and other Commissioners [of the NLRC] shall all
be appointed by the President, subject to confirmation by the Commission on Appointments (COA).

Pursuant to the said law, President Corazon Aquino appointed Bartolome Carale et al as the Chairman and
the Commissioners respectively of the NLRC. The appointments were however not submitted to the CoA for
its confirmation. Peter John Calderon questioned the appointment saying that without the confirmation by
the CoA, such an appointment is in violation of RA 6715. Calderon insisted that RA 6715 should be followed
as he asserted that RA 6715 is not an encroachment on the appointing power of the executive contained in
Sec. 16, Art. 7, of the Constitution, as Congress may, by law, require confirmation by the Commission on
Appointments of other officers appointed by the President in addition to those mentioned in the first
sentence of Sec. 16 of Article 7 of the Constitution.

ISSUE: Whether or not Congress may, by law, expand the list of public officers required to be confirmed by
the Commission on Appointment as listed in the Constitution.

HELD: No. Under the provisions of the 1987 Constitution, there are four (4) groups of officers whom the
President shall appoint. These four (4) groups are:

First, the heads of the executive departments, ambassadors, other public ministers and consuls, officers
of the armed forces from the rank of colonel or naval captain, and other officers whose appointments are
vested in him in this Constitution;
Second, all other officers of the Government whose appointments are not otherwise provided for by law;
Third, those whom the President may be authorized by law to appoint;
Fourth, officers lower in rank whose appointments the Congress may by law vest in the President alone.
The Supreme Court agreed with the Solicitor General: confirmation by the CoA is required exclusively for
the heads of executive departments, ambassadors, public ministers, consuls, officers of the armed forces
from the rank of colonel or naval captain, and other officers whose appointments are vested in the
President by the Constitution, such as the members of the various Constitutional Commissions (first
group). With respect to the other officers (second to fourth group) whose appointments are not otherwise
provided for by the law and to those whom the President may be authorized by law to appoint, no
confirmation by the Commission on Appointments is required.
Had it been the intention to allow Congress to expand the list of officers whose appointments must be
confirmed by the Commission on Appointments, the Constitution would have said so by adding the phrase
and other officers required by law at the end of the first sentence, or the phrase, with the consent of the
Commission on Appointments at the end of the second sentence. Evidently, our Constitution has
significantly omitted to provide for such additions.

This jurisprudence established the following in interpreting Sec 16, Art 7 of the Constitution
1. Confirmation by the Commission on Appointments is required only for presidential appointees
mentioned in the first sentence of Section 16, Article VII, including, those officers whose appointments are
expressly vested by the Constitution itself in the president (like sectoral representatives to Congress and
members of the constitutional commissions of Audit, Civil Service and Election).
2. Confirmation is not required when the President appoints other government officers whose
appointments are not otherwise provided for by law or those officers whom he may be authorized by law to
appoint (like the Chairman and Members of the Commission on Human Rights). Also, as observed in Mison,
when Congress creates inferior offices but omits to provide for appointment thereto, or provides in an
unconstitutional manner for such appointments, the officers are considered as among those whose
appointments are not otherwise provided for by law.

AYTONA VS CASTILLO
4 SCRA 1
FACTS:
Dominador Aytona was one of those appointed by outgoing president Carlos Garcia during the last day of
his term. Aytona was appointed as the ad interim governor of the Central Bank. When the next president,
Diosdado Macapagal took his office, he issued Order No. 2 which recalled Aytonas position and at the
same time he appointed Andres Castillo as the new governor of the Central Bank. Aytona then filed a quo
warranto proceeding claiming that he is qualified to remain as the Central Bank governor and that he was

validly appointed by the former president. Macapagal averred that the ex-presidents appointments were
scandalous, irregular, hurriedly done, contrary to law and the spirit of which, and it was an attempt to
subvert the incoming presidency or administration.

ISSUE: Whether or not Aytona should remain in his post.

HELD: No. Had the appointment of Aytona been done in good faith then he would have the right to
continue office. Here, even though Aytona is qualified to remain in his post as he is competent enough, his
appointment can nevertheless be revoked by the president. Garcias appointments are hurried maneuvers
to subvert the upcoming administration and is set to obstruct the policies of the next president. As a
general rule, once a person is qualified his appointment should not be revoked but in here it may be since
his appointment was grounded on bad faith, immorality and impropriety. In public service, it is not only
legality that is considered but also justice, fairness and righteousness.

JORGE VS MAYOR
10 SCRA 331
FACTS:
Nicanor G. Jorge, is a career official in the Bureau of Lands. He started working there as a Junior Computer
in the course of 38 years service, from February 1, 1922 to October 31, 1960, and attained the position of
Acting Director, through regular and successive promotions, in accordance with civil service rules. On June
17, 1961, he was designated Acting Director of the same Bureau, and on December 13, 1961 was
appointed by President Carlos Garcia ad interim Director. He qualified by taking the oath of office on the
December 23, 1961. His appointment was on December 26, 1961, transmitted to the Commission on
Appointments, and on May 14, 1962, petitioner's ad interim appointment as Director of Lands was
confirmed by the Commission.
On November 14, 1962 he received a letter from Benjamin Gozon, then Secretary of Agriculture and
Natural Resources of the Macapagal administration, informing him that pursuant to a letter from the
Assistant Executive Secretary Bernal, served on Jorge on November 13, his appointment was among those
revoked by Administrative Order No. 2 of President Diosdado Macapagal; that the position of Director of
Lands was considered vacant; and that Jorge was designated Acting Director of Lands, effective November
13, 1962. Upon learning that Mayor, an outsider, had been designate by the President to be Acting Director
of Lands Jorge protested (in a letter of November 16, 1962) to the Secretary of Agriculture informing the
latter that he would stand on his rights, and issued office circulars claiming to be the legally appointed
Director of Lands. Finally, on September 2, 1963, he instituted the present proceedings.
ISSUE: Whether or not Administrative Order No. 2 of President Macapaga operated as a valid revocation of
Jorge's ad interim appointment.
RULING:
No. The SC ruled that Nicanor G. Jorge is declared to be the duly appointed, confirmed, and qualified
Director of Lands.

Petitioner Jorge's ad interim appointment is dated December 13, 1961, but there is no evidence on record
that it was made and released after the joint session of Congress that ended on the same day. It is a
matter of contemporary history, of which SC may take judicial cognizance, that the session ended late in
the night of December 13, 1961, and, therefore, after regular office hours. In the absence of competent
evidence to the contrary, it is to be presumed that the appointment of Jorge was made before the close of
office hours, that being the regular course of business. The appointment, therefore, was not included in,
nor intended to be covered by, Administrative Order No. 2, and the same stands unrevoked. Consequently,
it was validly confirmed by the Commission on Appointments, and thereafter, the office never became
vacant.
In common with the Gillera appointment sustained by the SC, Jorge's appointment is featured by a
recognition of his tenure by the Macapagal administration itself, since he was allowed to hold and
discharge undisturbed his duties as de jure Director of Lands for nearly eleven months and it was only in
mid-November of 1962 that the attempt was actually made to demote him and appoint a rank outsider in
his place in the person of respondent Mayor.
If anyone is entitled to the protection of the civil service provisions of the Constitution, particularly those
against removals without lawful cause, it must be the officers who, like Jorge, entered the Civil Service in
their youth, bent on making a career out of it, gave it the best years of their lives and grew gray therein in
the hope and expectation that they would eventually attain the upper reaches and levels of the official
hierarchy, not through political patronage, but through loyalty, merit, and faithful and unremitting toil.

QUIMSING VS TAJANGLANGIT
G.R. No. L-19981 February 29 1964
FACTS:
May 16, 1962- Quimsings, as well as other peoples appointments were confirmed- May 17, 1962- at the
session of the Commission on Appointments, a motion for reconsideration of all the confirmed
appointments was approved, and the Commission was adjourned with no future date fixed for its next
meeting- June 11, 1962- President Macapagal designated Eduardo Tajanglangit as Acting Chief of Police of
Iloilo. Hence this petition for prohibition to restrain Eduardo Tajanglangit from occupying the position of
Chief of Police to which petitioner Quimsing had previously been appointed and duly qualified and the
functions of which he was actually discharging.

ISSUE: WON Quimsings appointment was not lawfully confirmed, because of the motion for
reconsideration of his confirmation, which has, to the present, remained unacted upon.

RULING:
The appointment of Tajanglangit to the position of Chief of Police of Iloilo City was null and void, because
said position was not vacant. The Revised Rules of the Commission on Appointments provide: SEC. 21:
Any motion to reconsider the vote on any appointment may be laid on the table, and this shall be final
disposition on such a motion." SEC. 22: Notice of confirmation or disapproval of an appointment shall not
be sent to the President of the Philippines before the expiration of the period for its reconsideration, or
while a motion for reconsideration is pending."
The Commission had not disapproved of Quimsings appointment, it was merely under reconsideration. It
has been established that on July 19, 1962, Quimsings appointment was delivered to Malacaang. This, as
well as the provisions above, supports the conclusion that the laying of a motion for reconsideration on the
table does not have the effect of withholding the effectivity of the confirmation, nor is it synonymous with
disapproval of the appointment. In fact, it is recognition that the appointment was confirmed.

ARTURO M. DE CASTRO vs. JUDICIAL AND BAR COUNCIL (JBC) and PRESIDENT GLORIA
MACAPAGAL ARROYO
G.R. No. 191002, March 17, 2010

FACTS:
The compulsory retirement of Chief Justice Reynato S. Puno by May 17, 2010 occurs just days after the
coming presidential elections on May 10, 2010.

These cases trace their genesis to the controversy that has arisen from the forthcoming compulsory
retirement of Chief Justice Puno on May 17, 2010, or seven days after the presidential election. Under
Section 4(1), in relation to Section 9, Article VIII, that vacancy shall be filled within ninety days from the
occurrence thereof from a list of at least three nominees prepared by the Judicial and Bar Council for
every vacancy. Also considering that Section 15, Article VII (Executive Department) of the Constitution
prohibits the President or Acting President from making appointments within two months immediately
before the next presidential elections and up to the end of his term, except temporary appointments to
executive positions when continued vacancies therein will prejudice public service or endanger public
safety.

The JBC, in its en banc meeting of January 18, 2010, unanimously agreed to start the process of filling up
the position of Chief Justice.
Conformably with its existing practice, the JBC automatically considered for the position of Chief Justice
the five most senior of the Associate Justices of the Court, namely: Associate Justice Antonio T. Carpio;
Associate Justice Renato C. Corona; Associate Justice Conchita Carpio Morales; Associate Justice Presbitero
J. Velasco, Jr.; and Associate Justice Antonio Eduardo B. Nachura. However, the last two declined their
nomination through letters dated January 18, 2010 and January 25, 2010, respectively.
The OSG contends that the incumbent President may appoint the next Chief Justice, because the
prohibition under Section 15, Article VII of the Constitution does not apply to appointments in the Supreme
Court. It argues that any vacancy in the Supreme Court must be filled within 90 days from its occurrence,
pursuant to Section 4(1), Article VIII of the Constitution; that had the framers intended the prohibition to
apply to Supreme Court appointments, they could have easily expressly stated so in the Constitution,
which explains why the prohibition found in Article VII (Executive Department) was not written in Article VIII
(Judicial Department); and that the framers also incorporated in Article VIII ample restrictions or limitations
on the Presidents power to appoint members of the Supreme Court to ensure its independence from
political vicissitudes and its insulation from political pressures, such as stringent qualifications for the
positions, the establishment of the JBC, the specified period within which the President shall appoint a
Supreme Court Justice.

A part of the question to be reviewed by the Court is whether the JBC properly initiated the process, there
being an insistence from some of the oppositors-intervenors that the JBC could only do so once the
vacancy has occurred (that is, after May 17, 2010). Another part is, of course, whether the JBC may resume
its process until the short list is prepared, in view of the provision of Section 4(1), Article VIII, which
unqualifiedly requires the President to appoint one from the short list to fill the vacancy in the Supreme
Court (be it the Chief Justice or an Associate Justice) within 90 days from the occurrence of the vacancy.

ISSUE: Whether the incumbent President can appoint the successor of Chief Justice Puno upon his
retirement.

HELD:
Prohibition under Section 15, Article VII does not apply to appointments to fill a vacancy in the Supreme
Court or to other appointments to the Judiciary.
Two constitutional provisions are seemingly in conflict.

The first, Section 15, Article VII (Executive Department), provides: Section 15. Two months immediately
before the next presidential elections and up to the end of his term, a President or Acting President shall
not make appointments, except temporary appointments to executive positions when continued vacancies
therein will prejudice public service or endanger public safety.
The other, Section 4 (1), Article VIII (Judicial Department), states: Section 4. (1). The Supreme Court shall
be composed of a Chief Justice and fourteen Associate Justices. It may sit en banc or in its discretion, in
division of three, five, or seven Members. Any vacancy shall be filled within ninety days from the
occurrence thereof.
Had the framers intended to extend the prohibition contained in Section 15, Article VII to the appointment
of Members of the Supreme Court, they could have explicitly done so. They could not have ignored the
meticulous ordering of the provisions. They would have easily and surely written the prohibition made
explicit in Section 15, Article VII as being equally applicable to the appointment of Members of the
Supreme Court in Article VIII itself, most likely in Section 4 (1), Article VIII. That such specification was not
done only reveals that the prohibition against the President or Acting President making appointments
within two months before the next presidential elections and up to the end of the Presidents or Acting
Presidents term does not refer to the Members of the Supreme Court.
Had the framers intended to extend the prohibition contained in Section 15, Article VII to the appointment
of Members of the Supreme Court, they could have explicitly done so. They could not have ignored the
meticulous ordering of the provisions. They would have easily and surely written the prohibition made
explicit in Section 15, Article VII as being equally applicable to the appointment of Members of the
Supreme Court in Article VIII itself, most likely in Section 4 (1), Article VIII. That such specification was not
done only reveals that the prohibition against the President or Acting President making appointments
within two months before the next presidential elections and up to the end of the Presidents or Acting
Presidents term does not refer to the Members of the Supreme Court.
Section 14, Section 15, and Section 16 are obviously of the same character, in that they affect the power
of the President to appoint. The fact that Section 14 and Section 16 refer only to appointments within the
Executive Department renders conclusive that Section 15 also applies only to the Executive Department.
This conclusion is consistent with the rule that every part of the statute must be interpreted with reference
to the context, i.e. that every part must be considered together with the other parts, and kept subservient
to the general intent of the whole enactment. It is absurd to assume that the framers deliberately situated
Section 15 between Section 14 and Section 16, if they intended Section 15 to cover all kinds of presidential
appointments. If that was their intention in respect of appointments to the Judiciary, the framers, if only to
be clear, would have easily and surely inserted a similar prohibition in Article VIII, most likely within Section
4 (1) thereof.

Separate Opinions
CARPIO MORALES, J., dissenting:
"Although the Chief Justice is primus inter pares, he cannot legally decide a case on his own because of the
Court's nature as a collegial body. Neither can the Chief Justice, by himself, overturn the decision of the
Court, whether of a division or the en banc."
Associate Justice Renato C. Corona in Complaint of Mr. Aurelio Indencia Arrienda against Justice
Puno, 499 Phil. 1, 14 (2005)

Primus Inter pares. First among equals. The Latin maxim indicates that a person is the most senior of a
group of people sharing the same rank or office. The phrase has been used to describe the status,

condition or role of the prime minister in most parliamentary nations, the high-ranking prelate in several
religious orders, and the chief justice in many supreme courts around the world.

The inclination to focus on the inter pares without due emphasis on the primus/prima has spawned
contemporary discourse that revives the original tug-of-war between domination and parity, which impasse
the conceived maxim precisely intended to resolve.

In the present case, several arguments attempt to depict a mirage of doomsday scenarios arising from the
impending vacancy of the primus in the Court as a springboard for their plea to avert a supposed
undermining of the independence of the judiciary. In reality, the essential question boils down to the
limitation on the appointing power of the President.

The ponencia of Justice Bersamin holds that the incumbent President can appoint the next Chief Justice
upon the retirement of Chief Justice Reynato S. Puno on May 17, 2010 since the prohibition during election
period does not extend to appointments in the judiciary, thereby reversing In re appointments of Hon.
Valenzuela & Hon. Vallarta.
The ponencia additionally holds that the Judicial and Bar Council (JBC) has until May 17, 2010, at the latest,
within which to submit to the President the list of nominees for the position of Chief Justice.
I DISSENT.

Constitutional draftsmanship style is the weakest aid in arriving at a constitutional


construction The first ratiocination adverts to the "organization and arrangement of the provisions of
the Constitution" that was, as the ponencia declares, purposely made by the framers of the Constitution to
"reflect their intention and manifest their vision" of the charter's contents.

It is unfortunate that the ponencia chiefly relies on the trivialities of draftsmanship style in arriving at a
constitutional construction. The petitioner in Anak Mindanao Party-List Group v. The Executive
Secretary raised a similar argument, but the Court held:
AMIN goes on to proffer the concept of "ordering the law" which, so it alleges, can be said of
the Constitution's distinct treatment of these three areas, as reflected in separate provisions in different
parts of the Constitution. It argues that the Constitution did not intend an over-arching concept of agrarian
reform to encompass the two other areas, and that how the law is ordered in a certain way should not be
undermined by mere executive orders in the guise of administrative efficiency.

The Court is not persuaded.

The interplay of various areas of reform in the promotion of social justice is not something implausible or
unlikely. Their interlocking nature cuts across labels and works against a rigid pigeonholing of executive

tasks among the members of the President's official family. Notably, the Constitution inhibited from
identifying and compartmentalizing the composition of the Cabinet. In vesting executive power in one
person rather than in a plural executive, the evident intention was to invest the power holder with energy.

AMIN takes premium on the severed treatment of these reform areas in marked provisions of
the Constitution. It is a precept, however, that inferences drawn from title, chapter or section
headings are entitled to very little weight. And so must reliance on sub-headings, or the lack
thereof, to support a strained deduction be given the weight of helium.

Secondary aids may be consulted to remove, not to create doubt. AMIN's thesis unsettles, more than
settles the order of things in construing the Constitution. Its interpretation fails to clearly establish
that the so-called "ordering" or arrangement of provisions in the Constitution was consciously
adopted to imply a signification in terms of government hierarchy from where a constitutional
mandate can per se be derived or asserted. It fails to demonstrate that the "ordering" or
layout was not simply a matter of style in constitutional drafting but one of intention in
government structuring. With its inherent ambiguity, the proposed interpretation cannot be made a
basis for declaring a law or governmental act unconstitutional. (emphasis and underscoring supplied)

Concededly, the allocation of three Articles in the Constitution devoted to the respective dynamics of the
three Departments was deliberately adopted by the framers to allocate the vast powers of government
among the three Departments in recognition of the principle of separation of powers.

The equation, however, does not end there. Such kind of formulation detaches itself from the concomitant
system of checks and balances. Section sequencing alone of Sections 14, 15 and 16 of Article VII, as
explained in the fourth ratiocination, does not suffice to signify functional structuring.

That the power of judicial appointment was lodged in the President is a recognized measure of limitation
on the power of the judiciary, which measure, however, is counterbalanced by the election ban due to the
need to insulate the judiciary from the political climate of presidential elections. To abandon this interplay
of checks and balances on the mere inference that the establishment of the JBC could de-politicize the
process of judicial appointments lacks constitutional mooring. The establishment of the JBC is not
sufficient to curtail the evils of midnight appointments in the judiciary.

The constitutional prohibition in Section 15 found its roots in the case of Aytona v.
Castillo, where among the "midnight" or "last minute" appointments voided to abort the abuse of
presidential prerogatives or partisan efforts to fill vacant positions were one in the Supreme Court and two
in the Court of Appeals.
Heeding Aytona's admonition, the Constitutional Commission (ConCom) saw it fit to provide for a
comprehensive ban on midnight appointments, finding that the establishment of the JBC is not enough to
safeguard or insulate judicial appointments from politicization. The ConCom deliberations reveal:

MR. GUINGONA: Madam President.

THE PRESIDENT: Commissioner Guingona is recognized.

MR. GUINGONA: Would the distinguished proponent accept an amendment to his amendment to limit this
prohibition to members of collegiate courts? The judges of the lower courts perhaps would not have the
same category or the same standing as the others mentioned here.

MR. DAVIDE: Pursuant to the post amendment, we already included here government-owned or controlled
corporations or their subsidiaries which are not even very sensitive positions. So with more reason that the
prohibition should apply to appointments in these bodies.

THE PRESIDENT: Does the Committee accept?

FR. BERNAS: What is common among these people Ministers, Deputy Ministers, heads of bureaus or
offices is that they are under the control of the President.

MR. GUINGONA: That is correct.


FR. BERNAS: Whereas, the other offices the Commissioner mentioned are independent offices.

MR. DAVIDE: The idea of the proposal is that about the end of the term of the President, he may prolong
his rule indirectly by appointing people to these sensitive positions, like the commissions, the Ombudsman,
the JUDICIARY, so he could perpetuate himself in power even beyond his term of office; therefore
foreclosing the right of his successor to make appointments to these positions. We should realize that the
term of the President is six years and under what we had voted on, there is no reelection for him. Yet he
can continue to rule the country through appointments made about the end of his term to these sensitive
positions.

FR. BERNAS: At any rate, there are other checks as far as the appointment of those officers is concerned.

MR. DAVIDE: Only insofar as the Commission on Appointments is concerned for offices which would require
consent, and the Judicial Bar Council insofar as the judiciary is concerned.

FR. BERNAS: We leave the matter to the body for a vote. (capitalization and emphasis supplied)

The clear intent of the framers is thus for the ban on midnight appointments to apply to the judiciary . The
succeeding interpellations suggest no departure from this intent.
For almost half a century, the seeds of Aytona, as nurtured and broadened by the Constitution, have grown
into an established doctrine that has weathered legal storms like Valenzuela.

The second ratiocination in the ponencia could thus not remove an added constitutional safeguard by
pretending to have examined and concluded that the establishment of the JBC had eliminated all
encompassing forms of political maneuverings during elections. Otherwise, reading into
the Constitution such conclusion so crucial to the scheme of checks and balances, which is neither written
nor tackled, undermines the noticeable silence or restraint exercised by the framers themselves from
making a definitive analysis.

To illustrate, the instance given in the fifth ratiocination that having the new President appoint the next
Chief Justice cannot ensure judicial independence because the appointee can also become beholden to the
appointing authority bears an inconsistent stance. It does not admit or recognize that the mechanism of
removal by impeachment eliminates the evils of political indebtedness. In any event, that level of
reasoning overlooks the risk of compromising judicial independence when the outgoing President faces the
Court in the charges that may be subsequently filed against her/him, and when the appointing President is
up for re-election in the peculiar situation contemplated by Section 4, Article VII of the Constitution.

All rules of statutory construction revolt against the interpretation arrived at by the ponencia

It is simplistic and unreliable for the ponencia to contend that had the framers intended to extend the ban
in Article VII to appointments in the judiciary, they would have easily and surely written so in Article VIII,
for it backlashes the question that had the framers intended to exclude judicial appointments in Article VIII
from the prohibition in Article VII, they would have easily and surely written so in the excepting proviso in
Article VII.

Taking into account how the framers painstakingly rummaged through various sections of
the Constitution and came up with only one exception with the need to specify the executive department,
it insults the collective intelligence and diligence of the ConCom to postulate that it intended to exclude
the judiciary but missed out on that one.

To hold that the ban on midnight appointments applies only to executive positions, and not to vacancies in
the judiciary and independent constitutional bodies, is to make the prohibition practically useless. It bears
noting that Section 15, Article VII of the Constitution already allows the President, by way of exception, to
make temporary appointments in the Executive Department during the prohibited period. Under this view,
there is virtually no restriction on the President's power of appointment during the prohibited period.

The general rule is clear since the prohibition applies to ALL kinds of midnight appointments.
The Constitution made no distinction. Ubi lex non distinguit nec nos distinguere debemos.

The exception is likewise clear. Expressio unius et exclusio alterius. The express mention of one person,
thing or consequence implies the exclusion of all others. There is no clear circumstance that would indicate
that the enumeration in the exception was not intended to be exclusive. Moreover, the fact that Section 15
was couched in negative language reinforces the exclusivity of the exception.

Under the rules of statutory construction, exceptions, as a general rule, should be strictly but reasonably
construed; they extend only so far as their language fairly warrants, and all doubts should be resolved in
favor of the general provisions rather than the exception. Where a general rule is established by statute
with exceptions, the court will not curtail the former nor add to the latter by implication . (italics in the
original; underscoring supplied)
The proclivity to innovate legal concepts is enticing. Lest the basic rule be forgotten, it helps to once more
recite that when the law is clear, it is not susceptible to interpretation and must be applied regardless of
who may be affected, even if the law may be harsh or onerous.

In its third ratiocination, the ponencia faults Valenzuela for not according weight and due consideration
to the opinion of Justice Florenz Regalado. It accords high regard to the opinion expressed by Justice
Regalado as a former ConCom Member, to the exception of the opinion of all others similarly situated.

It bears noting that the Court had spoken in one voice in Valenzuela. The ponencia should not hastily
reverse, on the sole basis of Justice Regalado's opinion, the Court's unanimous en banc decision penned by
Chief Justice Andres Narvasa, and concurred in by, inter alia, Associate Justices who later became Chief
Justices Hilario Davide, Jr., Artemio Panganiban and Reynato Puno.

The line of reasoning is specious. If that is the case and for accuracy's sake, we might as well reconvene all
ConCom members and put the matter to a vote among them.

Providentially, jurisprudence is replete with guiding principles to ascertain the true meaning of
the Constitution when the provisions as written appear unclear and the proceedings as
recorded provide little help:

While it is permissible in this jurisdiction to consult the debates and proceedings of the constitutional
convention in order to arrive at the reason and purpose of the resulting Constitution, resort thereto may be
had only when other guides fail as said proceedings are powerless to vary the terms of
the Constitution when the meaning is clear. Debates in the constitutional convention "are of value as
showing the views of the individual members, and as indicating the reasons for their votes, but they give
us no light as to the views of the large majority who did not talk, much less of the mass of our fellow
citizens whose votes at the polls gave that instrument the force of fundamental law. We think it safer to

construe the constitution from what appears upon its face." The proper interpretation therefore depends
more on how it was understood by the people adopting it than in the framers' understanding
thereof. (underscoring supplied) HAaDcS
The clear import of Section 15 of Article VII is readily apparent. The people may not be of the same caliber
as Justice Regalado, but they simply could not read into Section 15 something that is not there. Casus
omissus pro omisso habendus est.
What complicates the ponencia is its great preoccupation with Section 15 of Article VII, particularly its
fixation with sentences or phrases that are neither written nor referred to therein. Verba legis non est
recedendum, index animi sermo est. There should be no departure from the words of the statute, for
speech is the index of intention.

IN FINE, all rules of statutory construction virtually revolt against the interpretation arrived at by
the ponencia.
The 90-day period to fill a vacancy in the Supreme Court is suspended during the ban on
midnight appointments

Although practically there is no constitutional crisis or conflict involved upon the retirement of the
incumbent Chief Justice, the ponencia illustrates the inapplicability of the 90-day mandate to every
situation of vacancy in the Supreme Court (i.e., the 19-day vacuum articulated in the sixth ratiocination)
if only to buttress its thesis that judicial appointment is an exception to the midnight appointments ban.
The contemplated situation, however, supports the idea that the 90-day period is suspended during the
effectivity of the ban.

I submit that the more important and less complicated question is whether the 90-day period in Section 4
(1) of Article VIII runs during the period of prohibition in Section 15 of Article VII.

In response to that question, the ponencia declares that it is the President's "imperative duty to make an
appointment of a Member of the Supreme Court within 90 days from the occurrence of the vacancy [and
that t]he failure by the President to do so will be a clear disobedience to the Constitution."
The ponencia quotes certain records of the ConCom deliberations which, however, only support the view
that the number of Justices should "not be reduced for any appreciable length of time" and it is a "mandate
to the executive to fill the vacancy". Notably, there is no citation of any debate on how the framers
reckoned or determined an appreciable length of time of 90 days, in which case a delay of one day could
already bring about the evils it purports to avoid and spell a culpable violation of the Constitution. On the
contrary, that the addition of one month to the original proposal of 60 days was approved without
controversy ineluctably shows that the intent was not to strictly impose an inflexible timeframe.
Respecting the rationale for suspending the 90-day period, in cases where there is physical or legal
impossibility of compliance with the duty to fill the vacancy within the said period, the fulfillment of the
obligation is released because the law cannot exact compliance with what is impossible.

In the present case, there can only arise a legal impossibility when the JBC list is submitted or the
vacancy occurred during the appointments ban and the 90-day period would expire before the end of the
appointments ban, in which case the fresh 90-day period should start to run at noon of June 30. This was
the factual antecedent respecting the trial court judges involved in Valenzuela. There also arises a legal
impossibility when the list is submitted or the vacancy occurred prior to the ban and no appointment was
made before the ban starts, rendering the lapse of the 90-day period within the period of the ban, in which
case the remaining period should resume to run at noon of June 30. The outgoing President would be
released from non-fulfillment of the constitutional obligation, and the duty devolves upon the new
President.

Considering also that Section 15 of Article VII is an express limitation on the President's power of
appointment, the running of the 90-day period is deemed suspended during the period of the ban which
takes effect only once every six years.

This view differs from Valenzuela in that it does not implement Section 15 of Article VII so as to breach
Section 4 (1) of Article VIII. Instead of disregarding the 90-day period in the observance of the ban on
midnight appointments, the more logical reconciliation of the two subject provisions is to consider the ban
as having the effect of suspending the duty to make the appointment within 90 days from the occurrence
of the vacancy. Otherwise stated, since there is a ban, then there is no duty to appoint as the power to
appoint does not even exist. Accordingly, the 90-day period is suspended once the ban sets in and begins
or continues to run only upon the expiration of the ban.

One situation which could result in physical impossibility is the inability of the JBC to constitute a
quorum for some reasons beyond their control, as that depicted by Justice Arturo Brion in his Separate
Opinion, in which case the 90-day period could lapse without fulfilling the constitutional obligation.
Another such circumstance which could frustrate the ponencia's depiction of the inflexibility of the period
is a "no-takers" situation where, for some reason, there are no willing qualified nominees to become a
Member of the Court. Some might find this possibility remote, but then again, the situation at hand or the
"absurdity" of a 19-day overlapping vacuum may have also been perceived to be rare.

The seventh ratiocination is admittedly a non-issue. Suffice it to state that the Constitution is clear that
the appointment must come "from a list . . . prepared by the Judicial and Bar Council."

The Supreme Court can function effectively during the midnight appointments ban without an
appointed Chief Justice

The ponencia also holds that the JBC has until May 17, 2010, at the latest, within which to submit to the
President the list of nominees for the position of Chief Justice. It declares that the JBC should start the
process of selecting the candidates to fill the vacancy in the Supreme Court before the occurrence of the
vacancy, explaining that the 90-day period in the proviso, "Any vacancy shall be filled within ninety days
from the occurrence thereof," is addressed to the President, not to the JBC.

Such interpretation is absurd as it takes the application and nomination stages in isolation from the whole
appointment process. For the ponencia, the filling of the vacancy only involves the President, and the JBC
was not considered when the period was increased from 60 days to 90 days. The sense of the Concom is
the exact opposite.

The flaw in the reasoning is made more evident when the vacancy occurs by virtue of death of a member
of the Court. In that instance, the JBC could never anticipate the vacancy, and could never submit a list to
the President before the 90-day period.
Sustaining the view means that in case the President appoints as Chief Justice a sitting member of the
Court, from a JBC list which includes, for instance, incumbent justices and "outsiders," the JBC must
forthwith submit a list of nominees for the post left vacant by the sitting member-now new Chief Justice.
This thus calls for the JBC, in anticipation, to also commence and conclude another nomination process to
fill the vacancy, and simultaneously submit a list of nominees for such vacancy, together with the list of
nominees for the position of Chief Justice. If the President appoints an "outsider" like Sandiganbayan
Justice Edilberto Sandoval as Chief Justice, however, the JBC's toil and time in the second nomination
process are put to waste.
It is ironic for the ponencia to state on the one hand that the President would be deprived of ample time to
reflect on the qualifications of the nominees, and to show on the other hand that the President has, in
recent history, filled the vacancy in the position of Chief Justice in one or two days.
It is ironic for the ponencia to recognize that the President may need as much as 90 days of reflection in
appointing a member of the Court, and yet abhor the idea of an acting Chief Justice in the interregnum as
provided for by law, confirmed by tradition, and settled by jurisprudence to be an internal matter.

The express allowance of a 90-day period of vacancy rebuts any policy argument on the necessity to avoid
a vacuum of even a single day in the position of an appointed Chief Justice.

As a member of the Court, I strongly take exception to the ponencia's implication that the
Court cannot function without a sitting Chief Justice.
To begin with, judicial power is vested in one Supreme Court and not in its individual members, much less
in the Chief Justice alone. Notably, after Chief Justice Puno retires, the Court will have 14 members left,
which is more than sufficient to constitute a quorum.

The fundamental principle in the system of laws recognizes that there is only one Supreme Court from
whose decisions all other courts are required to take their bearings. While most of the Court's work is
performed by its three divisions, the Court remains one court single, unitary, complete and supreme.
Flowing from this is the fact that, while individual justices may dissent or only partially concur, when the
Court states what the law is, it speaks with only one voice.

The Court, as a collegial body, operates on a "one member, one vote" basis, whether it sits en banc or in
divisions. The competence, probity and independence of the Court en banc, or those of the Court's Division
to which the Chief Justice belongs, have never depended on whether the member voting as Chief Justice is
merely an acting Chief Justice or a duly appointed one.

IN LIGHT OF THE FOREGOING, I vote to hold, for the guidance of the Judicial and Bar Council, that the
incumbent President is constitutionally proscribed from appointing the successor of Chief Justice Reynato
S. Puno upon his retirement on May 17, 2010 until the ban ends at 12:00 noon of June 30, 2010.

GUVERA VS INOCENTES
16 Scra 379 1996 [Ad Interim Appointments]

FACTS:
On November 25, 1965, Onofre Guevara took his oath as an Undersecretary of Labor after his appointment
was extended ad interim on November 22, 1965. The appointment was questioned by Rauol Inocentes on
the ground that Guevaras appointment ceases to be valid after each term of Congress. At around midnight
of January 22, 1966, the Senate adjourned its session. The House of Representatives continued its session
and adjourned upon learning the Senates adjournment. In the case of Guevaras appointment, Congress,
through the Commission on appointments has not acted on it while the special session is being conducted.

ISSUE: Whether the ad interim appointment of Onofre P. Guevara is valid.


RULING:
Art. VII, Sec. 10, Subsection 4 of the 1935 Constitution: "the President shall have the power to make
appointments during the recess of the Congress, but such appointment shall be effective only until
disapproval by the Commission on Appointments or until the next adjournment of Congress"
The validity of an ad interim appointment shall be allowed when
(a) until disapproval of the Commission on Appointments and
(b) adjournment of Congress, whether special or regular session. In this case, the second mode of
termination took effect when the Congress adjourned sine die at about midnight of January 22,
1966 which made the appointment of petitioner Guevara ineffective.
The contention that the Commission on Appointments should be first organized before the second mode
can be made effective is untenable because they are two different and separate modes of termination.
Since the termination of ad interim appointment cannot be separated, the well-known maxim in statutory
construction applies. Ubi lex non distinguit nec nos distinguire debemus.
De Rama vs. CA

G.R. No. 131136, February 28, 2001

Facts: Upon his assumption to the position of Mayor of Pagbilao, Quezon, petitoner Conrado De Rama
wrote a letter to the CSC seeking the recall of the appointments of 14 municipal employees. Petitioner
justified his recall request on the allegation that the appointments of said employees were midnight
appointments of the former mayor, done in violation of Art. VII, Sec. 15 of the Constitution. The CSC denied
petitioners request for the recall of the appointments of the 14 employees for lack of merit. The CSC
dismissed petitioners allegation that these were midnight appointments, pointing out that the
constitutional provision relied upon by petitioner prohibits only those appointments made by an outgoing
President and cannot be made to apply to local elective officials. The CSC opined that the appointing
authority can validly issue appointments until his term has expired, as long as the appointee meets the
qualification
standards
for
the
position.
Issue: Whether or not the appointments made by the outgoing Mayor are forbidden under Art. VII, Sec. 15
of
the
Constitution
Held: The CSC correctly ruled that the constitutional prohibition on so-called midnight appointments,
specifically those made within 2 months immediately prior to the next presidential elections, applies only
to the President or Acting President. There is no law that prohibits local elective officials from making
appointments during the last days of his or her tenure.

MATIBAG VS. BENIPAYO


G.R. No. 149036, April 2, 2002
FACTS:
On February 1999, petitioner Matibag was appointed Acting Director IV of the Comelecs EID by then
Comelec Chairperson Harriet Demetriou in a temporary capacity. On March 2001, respondent Benipayo
was appointed Comelec Chairman together with other commissioners in an ad interim appointment. While
on such ad interim appointment, respondent Benipayo in his capacity as Chairman issued a Memorandum
address transferring petitioner to the Law Department. Petitioner requested Benipayo to reconsider her
relief as Director IV of the EID and her reassignment to the Law Department. She cited Civil Service
Commission Memorandum Circular No. 7 dated April 10, 2001, reminding heads of government offices that
"transfer and detail of employees are prohibited during the election period. Benipayo denied her request
for reconsideration on April 18, 2001, citing COMELEC Resolution No. 3300 dated November 6, 2000,
exempting Comelec from the coverage of the said Memo Circular.
Petitioner appealed the denial of her request for reconsideration to the COMELEC en banc. She also filed
an administrative and criminal complaint16 with the Law Department17against Benipayo, alleging that her
reassignment violated Section 261 (h) of the Omnibus Election Code, COMELEC Resolution No. 3258, Civil
Service Memorandum Circular No. 07, s. 001, and other pertinent administrative and civil service laws,
rules and regulations.
During the pendency of her complaint before the Law Department, petitioner filed the instant petition
questioning the appointment and the right to remain in office of Benipayo, Borra and Tuason, as Chairman
and Commissioners of the COMELEC, respectively. Petitioner claims that the ad interim appointments of
Benipayo, Borra and Tuason violate the constitutional provisions on the independence of the COMELEC.

ISSUES: Whether or not the assumption of office by Benipayo, Borra and Tuason on the basis of the ad
interim appointments issued by the President amounts to a temporary appointment prohibited by Section
1 (2), Article IX-C of the Constitution.

RULING:
We find petitioners argument without merit.
An ad interim appointment is a permanent appointment because it takes effect immediately and can no
longer be withdrawn by the President once the appointee has qualified into office. The fact that it is subject
to confirmation by the Commission on Appointments does not alter its permanent character. The
Constitution itself makes an ad interim appointment permanent in character by making it effective until
disapproved by the Commission on Appointments or until the next adjournment of Congress.
In the instant case, the President did in fact appoint permanent Commissioners to fill the vacancies in the
COMELEC, subject only to confirmation by the Commission on Appointments. Benipayo, Borra and Tuason
were extended permanent appointments during the recess of Congress. They were not appointed or
designated in a temporary or acting capacity, unlike Commissioner Haydee Yorac in Brillantes vs. Yorac34
and Solicitor General Felix Bautista in Nacionalista Party vs. Bautista.35 The ad interim appointments of
Benipayo, Borra and Tuason are expressly allowed by the Constitution which authorizes the President,
during the recess of Congress, to make appointments that take effect immediately.
While the Constitution mandates that the COMELEC "shall be independent"36, this provision should be
harmonized with the Presidents power to extend ad interim appointments. To hold that the independence
of the COMELEC requires the Commission on Appointments to first confirm ad interim appointees before
the appointees can assume office will negate the Presidents power to make ad interim appointments. This
is contrary to the rule on statutory construction to give meaning and effect to every provision of the law. It
will also run counter to the clear intent of the framers of the Constitution.
EMILIO A. GONZALES III, vs.OFFICE OF THE PRESIDENT OF THE PHILIPPINES, acting through and
represented by EXECUTIVE SECRETARY PAQUITO N. OCHOA, JR
G.R. No. 196231

September 4, 2012

x-----------------------x
WENDELL BARRERAS-SULIT, Petitioner, vs. OFFICE OF THE PRESIDENT.
PERLAS-BERNABE, J.:
G.R. No. 196232

These two petitions have been because they raise a common thread of issues relating to the President's
exercise of the power to remove from office herein petitioners who claim the protective cloak of
independence of the constitutionally-created office to which they belong - the Office of the Ombudsman.
The cases, G.R. No. 196231 and G.R. No. 196232 primarily seeks to declare as unconstitutional
Section 8(2) of Republic Act (R.A.) No. 6770, otherwise known as the Ombudsman Act of 1989,
which gives the President the power to dismiss a Deputy Ombudsman of the Office of the
Ombudsman.

FACTS:
G.R. No. 196231: A formal charge for Grave Misconduct (robbery, grave threats, robbery extortion and
physical injuries) was filed before PNP-NCR against Manila Police District Senior Inspector (P/S Insp.)
Rolando Mendoza and four others. Private complainant, Christian M. Kalaw, before the Office of the City
Prosecutor, filed a similar charge. While said cases were still pending, the Office of the Regional Director of
the National Police Commission (NPC) turned over, upon the request of petitioner Gonzales III, all relevant
documents and evidence in relation to said case to the Office of the Deputy Ombudsman for appropriate
administrative adjudication. Subsequently a case for Grave Misconduct was lodged against P/S Insp.
Rolando Mendoza and his fellow police officers in the Office of the Ombudsman. Meanwhile, the case filed
before the Office of the city Prosecutor was dismissed upon a finding that the material allegations made by
the complainant had not been substantiated "by any evidence at all to warrant the indictment of
respondents of the offenses charged." Similarly, the Internal Affairs Service of the PNP issued a Resolution
recommending the dismissal without prejudice of the administrative case against the same police officers,
for failure of the complainant to appear in three (3) consecutive hearings despite due notice. However,
upon the recommendation of petitioner Gonzales III, a Decision finding P/S Insp. Rolando Mendoza and his
fellow police officers guilty of Grave Misconduct was approved by the Ombudsman. Mendoza and his
colleagues filed for a motion for reconsideration which was forwarded to Ombudsman Gutierrez for final
approval, in whose office it remained pending for final review and action when P/S Insp. Mendoza hijacked
a bus-load of foreign tourists on that fateful day of August 23, 2010 in a desperate attempt to have himself
reinstated in the police service.
In the aftermath of the hostage-taking incident, which ended in the tragic murder of eight HongKong
Chinese nationals, the injury of seven others and the death of P/S Insp. Rolando Mendoza, a public outcry
against the blundering of government officials prompted the creation of the Incident Investigation and
Review Committee (IIRC). It was tasked to determine accountability for the incident through the conduct of
public hearings and executive sessions. The IIRC found Deputy Ombudsman Gonzales committed
serious and inexcusable negligence and gross violation of their own rules of procedure by
allowing Mendoza's motion for reconsideration to languish for more than nine (9) months
without any justification, in violation of the Ombudsman prescribed rules to resolve motions
for reconsideration in administrative disciplinary cases within five (5) days from submission.
The inaction is gross, considering there is no opposition thereto. The prolonged inaction precipitated the
desperate resort to hostage-taking. Petitioner was dismissed from service. Hence the petition.
G.R. No. 196232: Acting Deputy Special Prosecutor of the Office of the Ombudsman charged Major
General Carlos F. Garcia, his wife Clarita D. Garcia, their sons Ian Carl Garcia, Juan Paulo Garcia and
Timothy Mark Garcia and several unknown persons with Plunder and Money Laundering before the
Sandiganbayan. The Sandiganbayan denied Major General Garcia's urgent petition for bail holding that
strong prosecution evidence militated against the grant of bail. However, the government, represented by
petitioner, Special Prosecutor Barreras-Sulit and sought the Sandiganbayan's approval of a Plea Bargaining
Agreement ("PLEBARA") entered into with the accused. The Sandiganbayan issued a Resolution finding the
change of plea warranted and the PLEBARA compliant with jurisprudential guidelines.
Outraged by the backroom deal that could allow Major General Garcia to get off the hook with
nothing but a slap on the hand notwithstanding the prosecution's apparently strong evidence
of his culpability for serious public offenses, the House of Representatives' Committee on
Justice conducted public hearings on the PLEBARA. At the conclusion of these public hearings,
the Committee on Justice passed and adopted Committee Resolution No. 3, recommending to
the President the dismissal of petitioner Barreras-Sulit from the service and the filing of
appropriate charges against her Deputies and Assistants before the appropriate government
office for having committed acts and/or omissions tantamount to culpable violations of the
Constitution and betrayal of public trust, which are violations under the Anti-Graft and Corrupt
Practices Act and grounds for removal from office under the Ombudsman Act. Hence the petition.

ISSUE: Whether the Office of the President has jurisdiction to exercise administrative disciplinary power
over a Deputy Ombudsman and a Special Prosecutor who belong to the constitutionally-created Office of
the Ombudsman.

HELD:
YES. The Ombudsman's administrativedisciplinary power over a DeputyOmbudsman and
Special Prosecutor is not exclusive. While the Ombudsman's authority to discipline administratively is
extensive and covers all government officials, whether appointive or elective, with the exception only of
those officials removable by impeachment such authority is by no means exclusive. Petitioners cannot
insist that they should be solely and directly subject to the disciplinary authority of the Ombudsman. For,
while Section 21 of R.A. 6770 declares the Ombudsman's disciplinary authority over all government
officials, Section 8(2), on the other hand, grants the President express power of removal over a Deputy
Ombudsman and a Special Prosecutor. A harmonious construction of these two apparently conflicting
provisions in R.A. No. 6770 leads to the inevitable conclusion that Congress had intended the Ombudsman
and the President to exercise concurrent disciplinary jurisdiction over petitioners as Deputy Ombudsman
and Special Prosecutor, respectively. Indubitably, the manifest intent of Congress in enacting both
provisions - Section 8(2) and Section 21 - in the same Organic Act was to provide for an external authority,
through the person of the President, that would exercise the power of administrative discipline over the
Deputy Ombudsman and Special Prosecutor without in the least diminishing the constitutional and plenary
authority of the Ombudsman over all government officials and employees. Such legislative design is simply
a measure of "check and balance" intended to address the lawmakers' real and valid concern that the
Ombudsman and his Deputy may try to protect one another from administrative liabilities.
By granting express statutorypower to the President to removea Deputy Ombudsman and a
Special Prosecutor, Congressmerely filled an obvious gap inthe law. While the removal of the
Ombudsman himself is also expressly provided for in the Constitution, which is by impeachment under
Section 2 of the same Article, there is, however, no constitutional provision similarly dealing with the
removal from office of a Deputy Ombudsman, or a Special Prosecutor, for that matter. By enacting Section
8(2) of R.A. 6770, Congress simply filled a gap in the law without running afoul of any provision in the
Constitution or existing statutes. In fact, the Constitution itself, under Section 2, authorizes Congress to
provide for the removal of all other public officers, including the Deputy Ombudsman and Special
Prosecutor, who are not subject to impeachment.
The Power of the President toRemove a Deputy Ombudsmanand a Special Prosecutor isImplied
from his Power toAppoint. In giving the President the power to remove a Deputy Ombudsman and
Special Prosecutor, Congress simply laid down in express terms an authority that is already implied from
the President's constitutional authority to appoint the aforesaid officials in the Office of the Ombudsman.
The integrity and effectiveness of the Deputy Ombudsman for the MOLEO as a military watchdog looking
into abuses and irregularities that affect the general morale and professionalism in the military is certainly
of primordial importance in relation to the President's own role as Commander-in-Chief of the Armed
Forces. It would not be incongruous for Congress, therefore, to grant the President concurrent disciplinary
authority over the Deputy Ombudsman for the military and other law enforcement offices.
Granting the President the Powerto Remove a Deputy Ombudsmandoes not Diminish the
Independence of the Office of theOmbudsman. he claim that Section 8(2) of R.A. No. 6770 granting
the President the power to remove a Deputy Ombudsman from office totally frustrates, if not resultantly
negates the independence of the Office of the Ombudsman is tenuous. The independence which the Office
of the Ombudsman is vested with was intended to free it from political considerations in pursuing its
constitutional mandate to be a protector of the people. What the Constitution secures for the Office of the
Ombudsman is, essentially, political independence. This means nothing more than that "the terms of
office, the salary, the appointments and discipline of all persons under the office" are "reasonably insulated
from the whims of politicians."
Petitioner Gonzales may not beremoved from office where thequestioned acts, falling short of
constitutional standards, do notconstitute betrayal of public trust. Petitioner's act of directing the
PNP-IAS to endorse P/S Insp. Mendoza's case to the Ombudsman without citing any reason therefor cannot,

by itself, be considered a manifestation of his undue interest in the case that would amount to wrongful or
unlawful conduct. After all, taking cognizance of cases upon the request of concerned agencies or private
parties is part and parcel of the constitutional mandate of the Office of the Ombudsman to be the
"champion of the people." The factual circumstances that the case was turned over to the Office of the
Ombudsman upon petitioner's request; that administrative liability was pronounced against P/S Insp.
Mendoza even without the private complainant verifying the truth of his statements; that the decision was
immediately implemented; or that the motion for reconsideration thereof remained pending for more than
nine months cannot be simply taken as evidence of petitioner's undue interest in the case considering the
lack of evidence of any personal grudge, social ties or business affiliation with any of the parties to the
case that could have impelled him to act as he did. There was likewise no evidence at all of any bribery
that took place, or of any corrupt intention or questionable motivation. The OP's pronouncement of
administrative accountability against petitioner and the imposition upon him of the corresponding penalty
of dismissal must be reversed and set aside, as the findings of neglect of duty or misconduct in office do
not amount to a betrayal of public trust. Hence, the President, while he may be vested with authority,
cannot order the removal of petitioner as Deputy Ombudsman, there being no intentional wrongdoing of
the grave and serious kind amounting to a betrayal of public trust.
The Office of the President is vestedwith statutory authority to proceedadministratively
against petitionerBarreras-Sulit to determine theexistence of any of the grounds forher
removal from office as providedfor under the Constitution and theOmbudsman Act.
WHEREFORE, in G.R. No. 196231, the decision of the Office of the President in OP Case No. 10-J-460 is
REVERSED and SET ASIDE. Petitioner Emilio A. Gonzales III is ordered REINSTATED with payment of
backwages corresponding to the period of suspension effective immediately, even as the Office of the
Ombudsman is directed to proceed with the investigation in connection with the above case against
petitioner. In G.R. No. 196232, We AFFIRM the continuation of OP-DC Case No. 11-B-003 against Special
Prosecutor Wendell Barreras-Sulit for alleged acts and omissions tantamount to culpable violation of the
Constitution and a betrayal of public trust, in accordance with Section 8(2) of the Ombudsman Act of 1989.
The challenge to the constitutionality of Section 8(2) of the Ombudsman Act is hereby DENIED

Vous aimerez peut-être aussi